Past paper questions Flashcards

1
Q
Which of the following is NOT a potential cause of obstructive renal
impairment?
A Benign prostatic hypertrophy
B Recurrent kidney stones
C Retroperitoneal fibrosis
D Schistosomiasis
E Systemic sclerosis
A

Benign prostatic hypertrophy can cause urinary retention and increased pressure in the urinary outflow tract,

Fibrosis or ureter by recurrent kidney stones = obstruction.

Schistosomiasis granulomas forming around the eggs of schistosomes = obstruction.

Systemic sclerosis [condition affecting small blood vessels] leads to
fibrinoid thickening of the afferent arterioles, leading to reduced renal perfusion and thus renal impairment [ckd], but it does not cause an obstructive uropathy.

How well did you know this?
1
Not at all
2
3
4
5
Perfectly
2
Q

A 60-year-old man who works for an oil company presents with a lesion
on the temple that is bothering him as it is growing. It bled once when he
knocked it. On examination, the lesion is 8 mm in diameter and is a flat,
mildly erythematous patch with a few scales and a larger keratotic horn
in the centre. There are no other lesions on inspection of his skin and no
personal or family history of skin cancer.

Which of the following is the most appropriate management plan?
A Cryotherapy
B Curettage
C Excisional biopsy
D Topical 5-fl uorouracil
E Wide local excision
A

This lesion appears to be an actinic keratosis - Not at everted edges and well differentiated so unlikely to be SCC.

Treatment of isolated small lesions is by cryotherapy.
It has no concerning features to suggest that excision.

How well did you know this?
1
Not at all
2
3
4
5
Perfectly
3
Q

Which of the following best describes the MRI findings in multiple sclerosis?
A Cortical grey matter inflammatory lesions
B Longitudinally extensive transverse myelitis (more than three spinal
segments)
C Periventricular white matter lesions matching the clinical picture
D Periventricular white matter lesions not necessarily matching the clinical
picture
E White matter lesions exclusively in the cerebellum and brainstem

A

D Periventricular white matter lesions not necessarily matching the clinical picture.

This is because the central nervous system (CNS) inflammation does not
always cause demyelination or axonal damage of clinical significance, and
the CNS can recover from these foci of inflammation.

How well did you know this?
1
Not at all
2
3
4
5
Perfectly
4
Q

Which of the following is not a preventable risk factor for coronary artery
disease?
A Five cigarettes per day smoking history
B High low-density lipoprotein (LDL) cholesterol levels
C Hypertension
D Obesity
E 12 U/week alcohol history

A

12 U/week alcohol history

Low alcohol intake can reduce the risk.

How well did you know this?
1
Not at all
2
3
4
5
Perfectly
5
Q

A 55-year-old overweight pub landlord presents with a several-year history
of episodic acute painful joint swelling that started in his left big toe and
now affects his knees. Symptoms improve with use of diclofenac. Gout was
diagnosed on his first hospital visit, however this now appears recurrent.
He developed an acute attack in his left knee 2 days ago.
Which of the following represents the best plan for prophylaxis?
A Keep on long-term diclofenac with gastric protection
B Start allopurinol now with non-steroidal anti-inflammatory drugs
(NSAIDs) cover and increase until his urate is below 300 mmol/L
C Start allopurinol at least 2 weeks after the acute attack has settled with
NSAID cover and increase until his urate level is below 300 mmol/L
D Switch to long-term colchicine
E Switch to use of depot steroid injections

A

C Start allopurinol at least 2 weeks after the acute attack has settled with
NSAID cover and increase until his urate level is below 300 mmol/L.

Allopurinol can precipitate acute attacks so wait 2-4 weeks.

How well did you know this?
1
Not at all
2
3
4
5
Perfectly
6
Q

In which of the following circumstances should angiotensin-converting
enzyme (ACE) inhibitors be avoided where possible?
A Glomerulonephritis
B Lupus nephritis
C Renal artery stenosis
D Systemic sclerosis with renal involvement
E All of the above

A

Renal artery stenosis

Glomerulonephritis and lupus nephritis AND SYSTEMIC SCLEROSIS ACEI = renoprotection.

How well did you know this?
1
Not at all
2
3
4
5
Perfectly
7
Q

[43]Tetralogy of Fallot
An anxious mum has read on the internet about tetralogy of Fallot as she is
convinced her little boy may have it.
Which of the following does not fit the diagnosis?
A Her child is small for his age
B Her child is cyanotic
C His pulse exhibits a radio-femoral delay
D Her child exhibits a loud systolic murmur
E Her child can relive symptoms just by squatting

A

C His pulse exhibits a radio-femoral delay

Radio-femoral delay is a symptom of coarctation of the aorta, when the
coarctation (narrowing) occurs between the left subclavian artery and the aortic bifurcation.

Tetralogy of Fallot has four defining malformations:
1. Ventricular septal defect (VSD)
2. Pulmonary stenosis that turns the VSD into a right–left shunt, causing
cyanosis;
[acute attacks (due to increases in pulmonary vascular resistance)
can be managed by squatting, which raises peripheral vascular resistance
and therefore reduces the right–left shunt
3. Right ventricular hypertrophy
4. Overriding aorta, i.e. the aorta is connected to both ventricles and in this
sense acts like a second VSD, above which it sits

How well did you know this?
1
Not at all
2
3
4
5
Perfectly
8
Q

[44]Investigation of dysphagia
A 45-year-old man presents with intermittent difficulty in swallowing
for the last 4 months. This is associated with severe retrosternal pain and
regurgitation. He has no risk factors or sinister signs for malignancy.
What is the most important investigation in this case?
A Barium swallow
B Chest X-ray
C CT of the chest
D Endoscopy
E Iron studies

A

A - Achalsia - barium swallow is dx but

Dysphagia lasting more than 3 weeks, however, always warrants an endoscopy to exclude a malignant stricture.

How well did you know this?
1
Not at all
2
3
4
5
Perfectly
9
Q
[45] A 55-year-old Asian man with known thalassaemia trait registers with a
new GP and is found to have a mild microcytic anaemia on routine testing.
He does not complain of any symptoms.
What is the most appropriate treatment?
A Blood transfusion
B Folate supplementation
C Iron chelators
D Iron supplementation
E No treatment required
A

E

Individual Assymptomatic

How well did you know this?
1
Not at all
2
3
4
5
Perfectly
10
Q

A 72-year-old man is on warfarin for atrial fibrillation. Following a recent
chest infection his international normalised ratio (INR) rockets up to 5.2.
What was the most likely cause for this?
A Codeine phosphate
B Erythromycin
C Inappropriate high doses of warfarin
D International normalised ratio (INR) increased in concomitant infection
E Steroid inhalers

A

Erythromycin [potentiates the action of warfrin]

There is an interaction here with the antibiotics this woman is taking which has resulting in enhanced anticoagulation effects of warfarin

Cepahalosporins, chloramphenicol, ciprofloxacin, clarithryomycin, erythromycin and metronidazole are all examples which increase the effect of warfarin.

Any P450 inducer will have this effect as warfarin is a drug metabolised by cytochrome P450 enzymes. Antibiotics can also upset the gut flora which reduces vitamin K levels. [Remember that warfarin prevents the activation of vitamin K which is a cofactor in the synthesis of factors 2, 7, 9 and 10.]

How well did you know this?
1
Not at all
2
3
4
5
Perfectly
11
Q

Which of the following routine blood tests is most likely to indicate a cause
of an elderly patient’s acute confusional state?
A Calcium
B C-reactive protein (CRP)
C Liver function tests
D Sodium
E Urea

A

B- CRP

Infection = most common cause of acute confusional state in elderly.

How well did you know this?
1
Not at all
2
3
4
5
Perfectly
12
Q
Which of the following conditions is not associated with HLA-B27?
A Crohn’s disease
B Psoriasis
C Scleritis
D Ulcerative colitis
E Uveitis
A

C Scleritis

HLA-B27 Conditions
PEAR

Psoriatic arthritis
Enteropathic arthritis [IBD related]
Ankylosing spomdylitis
Reactive arthritis

How well did you know this?
1
Not at all
2
3
4
5
Perfectly
13
Q

[6] A 70-year-old man presents to the emergency department with a cough
productive of blood-stained green sputum and shortness of breath. A chest
X-ray demonstrates a suspicious lesion in the right lower zone associated
with consolidation. On further questioning, he admits to recent weight loss
and back pain. Blood tests show:

Corrected Ca2+ [high]
PO4 0.33 mmol/L {low}
Alk Phos 450 mmol/L

Which of the following is NOT appropriate in the management of this
patient?
A Check patient’s urea and electrolyte level and serum calcium level twice
daily
B Consider starting intravenous bisphosphonate
C Consider starting loop diuretics
D Rehydration with intravenous normal saline
E Urgent isotope bone scan is uncecessary.

A

Urgent isotope bone scan is uncecessary.

PTH like peptide from squamous cell cancer.

Consider starting loop diuretics - can aid renal clearance.

Intravenous bisphosphonate (e.g. pamidronate) could
cause a fall in calcium by causing bone reabsorption.
How well did you know this?
1
Not at all
2
3
4
5
Perfectly
14
Q

A 7-year-old boy presents with multiple erythematous patches, over both
sides of his face, head, neck, upper chest and left arm and shoulder, which
appear to be covered in a honey-coloured crust. His mother says that the
lesions have spread, over about a week, starting at the left neck and radiating
outwards. The child appears to be upset and the lesions are itchy. There is
no past medical history and no history of recent infections.
Which of the following diagnoses is most likely?
A Eczema
B Erysipelas
C Impetigo
D Psoriasis
E Staphylococcal scalded skin syndrome

A

impetigo - a sup[erficial infection by Staph Aureus that spreads from an initial starting point.

How well did you know this?
1
Not at all
2
3
4
5
Perfectly
15
Q

[21]A 69-year-old man recently diagnosed with metastatic prostate cancer
presents with weakness in his legs and urinary retention. He has had back
pain for years but in the last 24 hours this has become very severe in his
lower back. On examination he has a sensory deficit, loss of anal tone and
poor sensation in the skin around the anus. When catheterised he has a
residual volume of 1.5 L.
Which of the following is the most informative initial investigation?
A Computed tomography (CT) of the abdomen/pelvis
B Lumbar X-rays
C Magnetic resonance imaging (MRI) of the lumbar spine
D Serum calcium
E Ultrasound scan (USS) of the renal tract

A

Magnetic resonance imaging (MRI) of the lumbar spine - spinal chord compression

How well did you know this?
1
Not at all
2
3
4
5
Perfectly
16
Q

[21]A 69-year-old man recently diagnosed with metastatic prostate cancer
presents with weakness in his legs and urinary retention. He has had back
pain for years but in the last 24 hours this has become very severe in his
lower back. On examination he has a sensory deficit, loss of anal tone and
poor sensation in the skin around the anus. When catheterised he has a
residual volume of 1.5 L.
Which of the following is the most informative initial investigation?
A Computed tomography (CT) of the abdomen/pelvis
B Lumbar X-rays
C Magnetic resonance imaging (MRI) of the lumbar spine
D Serum calcium
E Ultrasound scan (USS) of the renal tract

A

Magnetic resonance imaging (MRI) of the lumbar spine - spinal chord compression.

Spinal chord compression =
lower Back pain with bowel/bladder/motor and sesnory dysfunction

[UMN signs]

How well did you know this?
1
Not at all
2
3
4
5
Perfectly
17
Q

[23]A 76-year-old man presents with a vesicular eruption on the left side of his
forehead only. It is severely painful and the vesicles have started to crust
over. On examination, the area affected is well-demarcated. You also note a
red eye with apparent conjunctivitis.
Given the most likely diagnosis, which of the following treatments is the
most appropriate?
A Intravenous aciclovir
B Oral aciclovir
C Topical aciclovir
D Topical antibiotic
E Topical steroids

A

B Oral aciclovir

Shingles - can affect branches of trigeminal nerve.

Most commonly the ophthalmic, where
it can also cause conjunctivitis, keratitis or iridocyclitis [Zoster opthalmicus]

How well did you know this?
1
Not at all
2
3
4
5
Perfectly
18
Q

[27] A 56-year-old man with a long history of alcohol abuse presents to the
emergency department with abdominal pain. On examination he has a
distended abdomen with shifting dullness and has a temperature of 38.2°C.
What is the most likely diagnosis?
A Bowel obstruction
B Liver cirrhosis
C Mallory–Weiss syndrome
D Perforated peptic ulcer
E Spontaneous bacterial peritonitis (SBP)

A

E Spontaneous bacterial peritonitis (SBP)

Patients with ascites are at risk of developing SBP, which usually presents
with severe generalised abdominal pain, worsening ascites, vomiting, fever
and rigors.

Intestinal luminal bacteria enter the circulation and colonise the ascitic fluid. SBP can lead to rapid decompensation of liver disease causing hepatic encephalopathy
and death.

The diagnosis of SBP is confirmed by paracentesis.

How well did you know this?
1
Not at all
2
3
4
5
Perfectly
19
Q

A 59-year-old woman with known polycythaemia vera presents to
the emergency department with right upper quadrant pain, tender
hepatomegaly and gross ascites, which has come on suddenly. There is no
jaundice.
What is the next most appropriate investigation?
A Cytomegalovirus (CMV) screen
B Gamma-glutamyl transferase (GGT) levels
C Hepatitis serology
D Hepatic vein Doppler ultrasound scan (USS)
E Human immunodeficiency virus (HIV) testing

A

Hepatic vein Doppler ultrasound scan (USS) -

Budd Chiari [sudden onset ascites, tender hepatomegaly + no jaundice due to blot clot obstructing hepatic portal vein - polycythaemia vera increased thrombosis risk in polycthaemia vera:
= stroke/heart attack/DVT/Budd-Chiari].

How well did you know this?
1
Not at all
2
3
4
5
Perfectly
20
Q

A 27-year-old man presents with a 3-month history of cough with
some blood streaking, loss of weight and night sweats. You suspect
tuberculosis.
Which of the following chest X-ray findings is not consistent with
tuberculosis?
A Cavitating lesion
B Consolidation of a lobe
C Diffuse 1–2 mm spots of increased opacity
D Perihilar ground-glass changes
E Pleural effusion

A

D Perihilar ground-glass changes [pulmonary fibrosis]

How well did you know this?
1
Not at all
2
3
4
5
Perfectly
21
Q

A 27-year-old man presents with a 3-month history of cough with
some blood streaking, loss of weight and night sweats. You suspect
tuberculosis.
Which of the following chest X-ray findings is not consistent with
tuberculosis?
A Cavitating lesion
B Consolidation of a lobe
C Diffuse 1–2 mm spots of increased opacity
D Perihilar ground-glass changes
E Pleural effusion

A

D Perihilar ground-glass changes [pulmonary fibrosis].

How well did you know this?
1
Not at all
2
3
4
5
Perfectly
22
Q

A 23-year-old man develops a urethral discharge and dysuria after a recent
change of sexual partner and urethral swabs are positive for Chlamydia.
Which of the following statements about antibiotic treatment is TRUE?
A His partners should be asked about symptoms and tested only if
symptomatic
B His partners should be tested for Chlamydia and treated only if positive
C His partners should be tested for Chlamydia and treated with antibiotics
regardless of outcome
D No antibiotic treatment is necessary provided he abstains from having
sexual intercourse for 2 weeks
E Only the patient who has presented needs antibiotic therapy

A

Common Sx of chlamydia = urethral discharge +

How well did you know this?
1
Not at all
2
3
4
5
Perfectly
23
Q

A 23-year-old man develops a urethral discharge and dysuria after a recent
change of sexual partner and urethral swabs are positive for Chlamydia.
Which of the following statements about antibiotic treatment is TRUE?
A His partners should be asked about symptoms and tested only if
symptomatic
B His partners should be tested for Chlamydia and treated only if positive
C His partners should be tested for Chlamydia and treated with antibiotics
regardless of outcome
D No antibiotic treatment is necessary provided he abstains from having
sexual intercourse for 2 weeks
E Only the patient who has presented needs antibiotic therapy

A

Common Sx of chlamydia = urethral discharge + dysuria [note same with ghonorrhoea - more discharge and yellow-green colour]

Pt can have absent Sx/mild sx that can resolve spontaneously - still be infectious so important to treat anyway [high likelihood].

RF - Epidymo-orchitis
Reactive arthritis
[Gonorrhoea = Rf for septic arthritis].

How well did you know this?
1
Not at all
2
3
4
5
Perfectly
24
Q

Which of the following is NOT a contraindication to renal transplantation?
A Active tuberculosis
B High-pressure urinary tract, e.g. posterior urethral valves
C Malignancy
D Severe arterial disease with stenosed iliac vessels
E Severe ischaemic heart disease with unstable angina and congestive
cardiac failure

A

Process of ellimination:
High-pressure urinary tract, e.g. posterior urethral valves

Active tuberculosis - Immunosuppression = disseminated TB.

Malignancy = Immunospuression worsen cancer/spread to transplant

Severe arterial disease with stenosed iliac vessels = Hypoperfucion = AKI.

Severe ischaemic heart disease with unstable angina and congestive
cardiac failure = Renal failure

How well did you know this?
1
Not at all
2
3
4
5
Perfectly
25
Q

[46]Thrombolysis in ischaemic stroke
Which of the following patients is eligible for thrombolysis with intravenous
recombinant tissue plasminogen activator?
A 2 hours post onset, BP 150/80 mmHg, GCS 11, MRI shows infarct
B 2 hours post onset, BP 160/90 mmHg, GCS 15, MRI shows haemorrhage
C 2 hours post onset, BP 160/95 mmHg, GCS 15, MRI shows infarct
D 2 hours post onset, BP 195/115 mmHg, GCS 15, MRI shows infarct
E 7 hours post onset, BP 135/80 mmHg, GCS 15, MRI shows infarct

A

Thrombolysis should only be done less than 4.5 hours from sx onset. [not E]

If greater than 4.5 hours, anti-coag with aspirin/clopidogrel 300mg.

Thrombectomy may be done on both cases.

Contra-indications of thrombolysis [anything that increases risk of bleed]:

  • Previous surgery past 3 months
  • On Anti-coagulation
  • Hemaorrhagic stroke/previous haemorrage
  • Very high BP [>180/110 mmHg]/low glascow scale [as increased likelihood that hemorragic]

Therefore answer is
C 2 hours post onset, BP 160/95 mmHg, GCS 15, MRI shows infarct

How well did you know this?
1
Not at all
2
3
4
5
Perfectly
26
Q

A 32-year-old man is diagnosed with Hodgkin’s lymphoma following a
recent history of weight loss and night sweats. Computed tomography
(CT) staging scan shows disease in the mediastinum bilaterally and some
abdominal lymphadenopathy, including the spleen, but no evidence of
disease in extranodal sites.
What is his stage of disease?
A Stage IIA
B Stage IIB
C Stage IIIA
D Stage IIIB
E Stage IVB

A

Ann arbour staging - Hodgkins lymphoma

Stage 1 = 1 lymph node
Stage 2 = 2+ ipsilaterally
Stage 3 = Bilateral lymph nodes
Stage 4 - Extra-nodal involvement

not A = Absence of B Sx
B = Presence of B Sx

How well did you know this?
1
Not at all
2
3
4
5
Perfectly
27
Q

[12] Which of the following scoring systems should be used to assess a patient’s
risk of developing a pressure score?
A Breslow score
B Confusion, Urea, Respiratory rate, Blood pressure, Age (CURB) score
C Ranson’s criteria
D Rockall score
E Waterlow score

A

Waterlow score

Breslow - Malignat melanoma
CURB 65 - pneumonia
Ranson’s - Pancreatitis
Rockall - upper GI bleed in PUD

How well did you know this?
1
Not at all
2
3
4
5
Perfectly
28
Q

A 35-year-old man presents with a 3-week history of pain and swelling in
the tips of his fingers. He has no history of bowel problems, recent infection
or skin disease, but his brother has Crohn’s disease. On examination you
find several swollen, red, tender distal interphalangeal joints, and the nails
have separated from the nailbed and have small pockmarks covering them.
The rest of the examination is unremarkable.
Which of the following diagnoses is most likely?
A Enteropathic arthritis
B Osteoarthritis
C Psoriatic arthropathy
D Reactive arthritis
E Rheumatoid arthritis

A

C Psoriatic arthropathy

"Brother has Crohn’s disease" - so likely to be HLA-B27 associated 
PEAR 
Psoriatic arthritis 
Enteropathic arthritis [IBD]
Ankylosing spondylitis 
Reactive arthritis 
Psoriatic  arthritis
Assymterical oligoarthritis
Symmetrical polyarthritis 
DIP joint predominence 
Nail changes [POSH]
Onycholysis [nail changes] means psoriatic arthritis most likely. 
Arthritis Muitilans [telescoping]

distal interphalangeal joints - so not Rheumatoid [PIP]

How well did you know this?
1
Not at all
2
3
4
5
Perfectly
29
Q
Which of the following is a recognised complication of coeliac disease?
A Fistulae
B Intestinal lymphoma
C Primary sclerosing cholangitis
D Toxic megacolon
E Uveitis
A

B Intestinal lymphoma
Complications of Coeliacs
T-cell lymphoma
Small bowel cancer

How well did you know this?
1
Not at all
2
3
4
5
Perfectly
30
Q

A 59-year-old woman with advanced metastatic breast cancer presents to
the emergency department with severe abdominal pain. She has not opened
her bowels for 7 days and feels constipated. She has also noticed that she
has been passing water more frequently but has not been incontinent. On
rectal examination there is no loss of anal tone and normal sensation.
What is the most likely diagnosis?
A Hypercalcaemia
B Hypocalcaemia
C Metastatic spread to the bowel
D Opiate-induced constipation
E Spinal cord compression

A

Hypercalcaemia - secondary to bone mets

Stones, bones, groans, thrones, psychic moans

How well did you know this?
1
Not at all
2
3
4
5
Perfectly
31
Q

Pt has DIC secondary to Meningococcal sepsis:

Which of the following investigation results would you NOT expect in
disseminated intravascular coagulation?
A Increased activated partial thromboplastin time (APTT)
B Increased fibrinogen
C Increased international normalised ratio (INR)
D Decreased haemoglobin
E Decreased platelets

A

B Increased fibrinogen - Fibrinogen used in clot formation so will be depleated.

How well did you know this?
1
Not at all
2
3
4
5
Perfectly
32
Q

A 70-year-old woman who has been in hospital for 5 days with severe
pneumonia and treated successfully with intravenous co-amoxiclav and
oral clarithromycin develops profuse watery diarrhoea. Her pulse rises
to 110 bpm and she develops a temperature of 38.3°C. She is also taking
omeprazole for reflux disease. Stool samples are positive for Clostridium
difficile toxin.
Which one of the following is the best treatment plan?
A Barrier nursing in a side room
B Continue present antibiotic therapy
C Intravenous vancomycin
D Oral vancomycin
E Stop co-amoxiclav and omeprazole

A

C.did treatment = Ora; vancomycin/metronidazole

Note - Vancomycin used for MRSA also.

How well did you know this?
1
Not at all
2
3
4
5
Perfectly
33
Q

A 34-year-old woman with systemic lupus erythematosus (SLE) has had
multiple miscarriages and now presents with a painful right swollen leg. A
compression ultrasound scan confirms deep vein thrombosis.
Which blood test may now be indicated?
A Anti-phospholipid antibodies
B Clotting factors
C Haemoglobin
D Pregnancy test
E Tumour markers

A

Anti-phospholipid antibodies

Anti-phospholipd syndrome is characterised by:
Presence of anti-phospholipid antibodies
Pro-thrombotic state [arterial + Venous] and thrombocytopaenia
Recurrant miscarriages

Ass w/SLE

How well did you know this?
1
Not at all
2
3
4
5
Perfectly
34
Q

A 42-year-old woman attends to her GP complaining of non-specific
abdominal pain and an increasing abdominal girth. She is found to have
a large mass in her right lower abdomen and ascites on transvaginal
ultrasound imaging.
Which of the following tumour markers would be most useful?
A CA 125
B Ca 15-3
C Ca 19-9
D CEA
E Beta-hCG

A

CA 125 - Ovarian Cancer [FLAWS + abdo pain + abdo girth + ascites]

How well did you know this?
1
Not at all
2
3
4
5
Perfectly
35
Q

[21] You are asked by your registrar to see a 40-year-old woman and report back
your findings. On examination, you struggle to find an apex beat although
heart sounds 1 and 2 were audible with no murmur. On inspection, her
electrocardiogram (ECG) is normal except for inverted P-waves.
What is the most likely reason for these findings?
A Dextrocardia
B Cardiomyopathy
C Mitral stenosis
D Myocardial ischaemia
E Pulmonary hypertension

A

Dextrocardia

Inverted P-waves can be caused by dextrocardia or by the natural cardiac
pacemaker being situated elsewhere in the atrium other than the sino-atrial
node. In dextrocardia, the apex beat would be palpable on the right. The
ECG leads should be reversed to yield a “normal” trace.

How well did you know this?
1
Not at all
2
3
4
5
Perfectly
36
Q

[q1] You find an 80-year-old man collapsed in the street. He is unresponsive
and is making a snoring sound. An ambulance has been called but has yet
to arrive.
Which of the following is the best course of action?
A Cricothyroidotomy
B Do nothing till the ambulance arrives
C Finger sweep
D Head tilt chin lift
E Place in the recovery position

A

Head tilt chin lift - secure airway first

ABC

How well did you know this?
1
Not at all
2
3
4
5
Perfectly
37
Q

A 55-year-old man is being investigated for irregular heart rhythms. He has
a medical history of diabetes mellitus. He explains that exercise is diffi cult
for him due to joint pains. During the examination it is noted that he has
tan skin pigmentation and hepatomegaly.
Which of the following investigations could reveal the aetiology of his
symptoms?
A Haematinics
B Serum caeruloplasmin
C Short synacthen test
D Alpha-1 antitrypsin
E Gamma-GT

A

Haematinics - Haemochromatosis
{iron deposition in pancreas = DM
In joints = Koint pain
In heart = arrhythmias due to cardiomyopathy

Increased pigmentation - Fe in skin.

How well did you know this?
1
Not at all
2
3
4
5
Perfectly
38
Q

A 66-year-old man with a 10-year history of chronic obstructive pulmonary
disease is assessed in the respiratory clinic for eligibility for long-term
domiciliary oxygen therapy.
Which of the following is NOT a criterion for prescription of long-term
oxygen therapy?
A No exacerbation of chronic obstructive pulmonary disease (COPD) for
the previous 5 weeks
B Patient has stopped smoking
C Patient has chronic hypoxaemia with PaO2 <7.3 kPa
D Presence of pulmonary hypertension with PaO2 <8.0 kPa
E Two arterial blood gases showing PaO2 <7.3 kPa within 7 days

A

Two arterial blood gases showing PaO2 <7.3 kPa within 7 days

Must be >3 weeks apart

Criteria for long term O2 therapy: 
 PaO2 of <7.3kpa [measured 3 weeks apart]
PaO2 - 7.3 -8 w/
Pulm HyperT
Secondary polycthaemia 
Peripheral odema 
Nocturnal hypoxia 

No exacerbation of chronic obstructive pulmonary disease (COPD) for
the previous 5 weeks

How well did you know this?
1
Not at all
2
3
4
5
Perfectly
39
Q

A 35-year-old alcoholic homeless man with status epilepticus - management
A Diazepam 2 mg intravenously
B Diazepam 2 mg intravenously and Pabrinex intravenously
C Lorazepam 4 mg intravenously and Pabrinex intravenously
D Lorazepam 8 mg intravenously
E Lorazepam 8 mg intravenously and Pabrinex intravenously

A

4mg Lorazepam
or
10mg Diazepam

(+IV pabrinex)

How well did you know this?
1
Not at all
2
3
4
5
Perfectly
40
Q

A 55-year-old man presents to his GP with increasing lethargy and
polyuria. He has a past medical history of ischaemic heart disease and
congestive cardiac failure. He smokes 30 cigarettes per day and drinks
alcohol occasionally. He has a body mass index (BMI) of 32. His random
blood glucose is 14.0 mmol/L and fasting blood glucose level is 9.0
mmol/L.
Which of the following management is NOT appropriate in this patient?
A Advise the patient to change his diet and stop smoking
B Metformin should be considered as the first-line oral treatment option
for overweight patients
C Sulphonylureas and metformin could be considered as a combined
therapy if glycaemic control is not optimal
D Sulphonylureas should be considered if patient is intolerant to metformin
E Thiazolidinediones can be added to metformin and sulphonylurea
combination therapy if control is not optimal

A

E Thiazolidinediones can be added to metformin and sulphonylurea
combination therapy if control is not optimal.

Thiazolidinediones = acts on the adipocytes and addresses insulin resistance peripherally in fat and muscle [increases glucose uptake] but increase NA+ absorption = increasing fluid so contarindicated in CKD and CCF.

GLP-1 agonists and DDP4 inhibirtors - incretin effect * It stimulates insulin and suppresses glucagon
* It also increases satiety

  • Metformin = Treat insulin resistance in the liver - It is an insulin sensitiser - reduces hepatic glucose output + = wt loss
  • Sulphonylureas = it makes the existing pancreas secrete more insulin
  • Alpha glucosidase inhibitor = delays glucose absorption [helps with 1st phase insulin defect]
  • Thiazolidinediones = acts on the adipocytes and addresses insulin resistance peripherally in fat and muscle [increases glucose uptake].
  • SGLT2 inhibitors - increase glucose excretion by the kidneys.
How well did you know this?
1
Not at all
2
3
4
5
Perfectly
41
Q
Which of the below results is the best indicator of poor liver function?
A Raised alanine transferase
B Raised albumin
C Raised alkaline phosphatase
D Raised bilirubin
E Prolonged prothrombin time
A

E Prolonged prothrombin time - secondary to reduced synthetic ft of liver = reduces clotting factors.

How well did you know this?
1
Not at all
2
3
4
5
Perfectly
42
Q

[22] He will need physiological
fluid replacement when he is nil-by-mouth. He weighs 70 kg.
Which of the following regimens is closest to physiological needs?

A 1 L 0.9% normal saline with 20 mmol potassium and 2 × 1 L 5% dextrose
in 24 hours
B 1 L 0.9% normal saline with 20 mmol potassium and 2 × 1 L 5% dextrose
with 20 mmol potassium in 24 hours
C 2 × 1 L 0.9% normal saline with 20 mmol potassium and 1 L 5% dextrose
in 24 hours
D 2 × 1 L 0.9% normal saline with 20 mmol potassium and 1 L 5% dextrose
with 20 mmol potassium in 24 hours
E 3 L Hartmann’s in 24 hours

A

L 0.9% normal saline with 20 mmol potassium and 2x 1 L 5%
dextrose with 20 mmol potassium in 24 hours

A man weighing 70 kg needs 3L of water a day.

140mmol of Na+
60mmol K+

Normal saline contains 155 mmol of sodium.

Hartmann’s contains 130
mmol of sodium and 5 mmol of potassium, as well as chloride, calcium and
lactate. 3 L of Hartmann’s will give too much sodium and not
enough potassium, which could lead to fluid retention and hypokalaemia

How well did you know this?
1
Not at all
2
3
4
5
Perfectly
43
Q
[25] Which of the following diseases is the most common reason for misdiagnosis
of Parkinson’s disease?
A Corticobasal degeneration
B Essential tremor/familial tremor
C Progressive supranuclear palsy
D Pugilist encephalopathy
E Wilson’s disease
A

Essential tremor/familial tremor
[No other parkinsonian symptoms are present]
Sx improvement with beta blockers.

Usually the earliest sign of Parkinson’s disease,
and essential/familial tremor can present in late middle age, frequent
misdiagnosis is understandable. Essential and familial tremor are similar,
except essential tremor has no family history.

Progressive supranuclear palsy = parkinsonian symptoms with a
supranuclear gaze palsy
[A supranuclear gaze palsy is an inability to look in a particular direction as a result of cerebral impairment. There is a loss of the voluntary aspect of eye movements].

How well did you know this?
1
Not at all
2
3
4
5
Perfectly
44
Q

[38] A 24-year-old woman who has been travelling to India on a gap year
presents to clinic as she is concerned about a “funny-looking mole” on her
leg. She is unsure how long it has been there.
Which of the following is not a concerning feature of a mole when
considering a diagnosis of malignant melanoma?
A Asymmetry
B Bleeding
C Border irregularity
D Colour different to that of other moles on patient
E Itching

A

Colour different to that of other moles on patient

How well did you know this?
1
Not at all
2
3
4
5
Perfectly
45
Q

[37] A 74-year-old woman presents to hospital with an acute right-sided
hemiparesis, and is found to have a left middle cerebral artery infarct
on diffusion-weighted magnetic resonance imaging (MRI). It is her first
stroke. Her past medical history is unremarkable. Her blood pressure
is normal and her electrocardiogram (ECG) shows sinus rhythm with
occasional ventricular ectopics. Blood tests show normal cholesterol and
normal glucose. On carotid Doppler she is found to have an 85% stenosis
of the left carotid.
Which of the following treatments will NOT benefit her?
A ACE-inhibitor
B Aspirin
C Left carotid endarterectomy
D Statin
E Warfarin

A

E Warfarin

Warfarin is not indicated here as there is no atrial fibrillation,
it is her first stroke and there is nothing sinister in the past medical history
suggesting a drastic predisposition to thrombus formation. So Aspirin os sufficient anti-coag.

ACE-inhibitor
therapy has a beneficial effect even if the blood pressure was previously
normal.

How well did you know this?
1
Not at all
2
3
4
5
Perfectly
46
Q

[38] A 24-year-old woman who has been travelling to India on a gap year
presents to clinic as she is concerned about a “funny-looking mole” on her
leg. She is unsure how long it has been there.
Which of the following is not a concerning feature of a mole when
considering a diagnosis of malignant melanoma?
A Asymmetry
B Bleeding
C Border irregularity
D Colour different to that of other moles on patient
E Itching

A

Colour different to that of other moles on patient

ABCDE

Assymetry 
Border Irregularity 
Colour variation within smae mole 
Diameter [>7+mm]
Evolution of lesion [size, shape,
colour)

Other Sx - inflammation, bleading/itching/erythema et.c

How well did you know this?
1
Not at all
2
3
4
5
Perfectly
47
Q
A 43-year-old woman attends the GP with a 3-month history of a grey–white
vaginal discharge which she says has a “fishy” odour. She is systemically
well and has no menstrual abnormalities.
What is the most likely diagnosis?
A Bacterial vaginosis (BV)
B Candida
C Chlamydia
D Gonorrhoea
E Syphilis
A

Grey-white discharge w/fishy odour is classically bacterial vaginosis [not STI].

How well did you know this?
1
Not at all
2
3
4
5
Perfectly
48
Q

[43] An 83-year-old man is admitted with acute confusion. He has an extensive
medical history including atrial fibrillation, type 2 diabetes, osteoarthritis,
hypertension and some mild congestive cardiac failure, for which he takes
several medications. He appears clinically dry, with a pulse of 115/min, dry
mucous membranes and a capillary refill rate of 4 seconds. He is noted
to have a reduced urine output with concentrated urine. His creatinine is
235 μmol/L.
Which of the following medications does not need be reduced or stopped?
A Amlodipine
B Diclofenac
C Digoxin
D Furosemide
E Metformin

A

Presentaion suggests AKI - oligouria + relevant RFs.

calcium-channel blocker, a = safest antihypertensives for kidneys.

NSAIDs = nephrotoxic [interfere with effecternt arteriole = hypoperfusion of kidney]

Digoxin = nephrotoxic

Furosemide = exacerbation of hypovolaemia

Metformin = risk of met acidosis [hypoglycaemia] so should be stopped.

How well did you know this?
1
Not at all
2
3
4
5
Perfectly
49
Q

[50] A 30-year-old man presents to the medical assessment unit with a history
of excessive drinking and urination. He has been going to the toilet about
7–8 times per day for 1 month. His
results on admission show:

Urine osmolality 145 mOsm/kg
Water deprivation test – urine osmolality 296 mOsm/kg after DDAVP 2 μg
administered intramuscularly
What is the most likely diagnosis?
A Acute tubular necrosis
B Cranial diabetes insipidus
C Nephrogenic diabetes insipidus
D Primary polydipsia
E Type 2 diabetes mellitus
A

ADH results in increased urine osmolarity ∴ cranial diabetes insipidus.

How well did you know this?
1
Not at all
2
3
4
5
Perfectly
50
Q

A 60-year-old man visits his GP complaining of tiredness. He has noticed weight
loss over the last six months and irritation of the tip of his penis which appears
inflamed on examination. He mentions he has been visiting the toilet more often
than usual and feeling thirsty. The most appropriate investigation would be:
A. Oral glucose tolerance test
B. Measurement of glycated haemoglobin
C. Random plasma glucose test
D. Water deprivation test
E. Measurement of triglyceride levels

A

T2DM - Polydyspia, polyuria and wt loss

Opportunistic infection w/candida in pts with diabetes:
Females - Pruritius Vulvae
Males - Penile inflammation Balanitis

How well did you know this?
1
Not at all
2
3
4
5
Perfectly
51
Q

A 28-year-old woman has noticed a change in her appearance; most notably her
clothes do not fit properly and are especially tight around the waist. Her face
appears flushed and more rounded than usual, despite exercising regularly and
eating healthily her weight has steadily increased over the last 3 weeks. On visiting
her GP, he notices her blood pressure has increased since her last visit and she has
bruises on her arm. She is especially worried about a brain tumour. The next most
appropriate investigation would be:
A. Low-dose dexamethasone test
B. High-dose dexamethasone test
C. Urinary catecholamines
D. Computed tomography (CT) scan
E. Urinary free cortisol measurement

A

Urinary free cortisol [1st]

Then low dose dexamathasone in pt with Cushings.

How well did you know this?
1
Not at all
2
3
4
5
Perfectly
52
Q

A 55-year-old diabetic woman presents with altered sensations in her hands and
feet. She finds it difficult to turn pages of books and discriminating between
different coins. When walking, the floor feels different and she likens the sensation
to walking on cotton wool. The most likely diagnosis is:
A. Autonomic neuropathy
B. Diabetic amyotrophy
C. Acute painful neuropathy
D. Symmetrical sensory neuropathy
E. Diabetic mononeuropathy

A

Answer = Symmetrical sensory neuropathy - gloves and stocking distribution

Diabetic Neuropathy - weakness, reduced sensation and burning/shooting pain in known diabetic.
[tends not to be suymmetrical]

Diabetic amyotrophy - proximal thigh muscle Pain weakness and atrophy [quads]

Autonomic neuropathy - subtype of diabetic neuropathy = Autonomic sx [e.g postural hypotension, ED, bowel/bladder dysfunction, sweating anxiety]

How well did you know this?
1
Not at all
2
3
4
5
Perfectly
53
Q

A 38-year-old woman presents to clinic complaining of changes in her appearance
and weight gain. She has recently been through a divorce and attributed her weight
gain to this. However, despite going to the gym her clothes are still tight, especially
around her waist, her face seems puffy and flushed. The most likely diagnosis is:
A. Hyperthyroidism
B. Cushing’s disease
C. Acromegaly
D. Hypothyroidism
E. Diabetes

A

Facial plethora is a Sx of acromegaly alongside othger features such as coarse facial faeatures, prognathism, carpall tunnel, OSA …

Acromegaly often co-presents with hyperprolactinaemia.

How well did you know this?
1
Not at all
2
3
4
5
Perfectly
54
Q
A 67-year-old man is brought into A&amp;E having been involved in a road traffic accident. On examination, he opens his eyes to pain, makes a few grunting noises and withdraws his legs from painful stimuli. What is his GCS?
A 2
B 4
C 6
D 8
E 10
A

E= 2
V=2
M=4
Answer = 8

Eyes [1-4]

Verbal [1-5]
1 = no sound 
2 = few incomprehensible sounds 
3 = inappropriate responses 
4= confused conversation
5= oriented in time and space 
Motor [1-6]
1= no movement 
2= abnormal extension 
3=  abnormal flexion 
4= withdraws from pain 
5= Moves to painful stimulus
6=  obeys command for movement
How well did you know this?
1
Not at all
2
3
4
5
Perfectly
55
Q

56-year-old man with a history of alcoholism complains of intermittent epigastric pain that radiates through to his back. When questioned, he admits to losing about 3 kg in weight over the past 6 months and says that his stools have become pale and difficult to flush away. Which investigation would you request to aid the diagnosis?

A Serum amylase
B Blood cultures
C Faecal elastase
D CA 19-9
E OGD
A

Chronic pancreatitis
C Faecal elastase = best marker for chronic pancreatitis

[amylase/lipase may not be raised due to exocrine insufficency]

How well did you know this?
1
Not at all
2
3
4
5
Perfectly
56
Q

A 46-year-old man, with a history of type 1 diabetes, visits the GP for an HbA1c reading. He has recently been feeling more tired than usual and has noticed that the skin on his hands has become darker over the past few months. On examination, hepatomegaly and a tanned complexion (despite not having been on any recent holidays) are noted. Haemochromatosis is suspected and iron studies are requested. Which set of results would be consistent with haemochromatosis?
A High serum iron, high ferritin, high transferrin, low transferrin saturation, low TIBC
B High serum iron, low ferritin, low transferrin, high transferrin saturation, low TIBC
C High serum iron, high ferritin, high transferrin, high transferrin saturation, low TIBC
D High serum iron, high ferritin, low transferrin, high transferrin saturation, high TIBC
E High

A

E

Haemochromatosis = 
Hereditary haemochromatosis (HH) is an autosomal recessive disorder of iron metabolism caused by excessive intestinal absorption of dietary iron. This results in iron deposition in tissues and organs (e.g. skin, joints, liver, pancreas and adrenal glands).

The main clinical features of HH are bronze skin, diabetes and hepatomegaly, due to iron deposition in the skin, pancreas and liver

High serum iron
Increased ferritin [responsible for intracellular storage of iron]
Low TIBC
High transferrin saturation [Fe2+ binds transferrin in blood]
Low Transferrin – increased serum iron leads to decreased transferrin levels to prevent more iron from becoming plasma protein bound in the blood

How well did you know this?
1
Not at all
2
3
4
5
Perfectly
57
Q

A 53-year-old man presents to A&E with severe pain in his right flank that radiates to his right groin. Ureteric colic is suspected and a CT-KUB is requested. The CT-KUB confirms the diagnosis but it also shows an abdominal aortic aneurysm with a diameter of 4.7 cm. When questioned, the patient denies any back pain (other than the pain caused by ureteric colic) or symptoms of vascular disease. What is the most appropriate management option for this patient?
A Reassure and discharge
B Surveillance with an ultrasound scan every 1 year
C Surveillance with an ultrasound scan every 6 months
D Surveillance with an ultrasound scan every 3 months
E Surgical repair of the aneurysm

A

D Surveillance with an ultrasound scan every 3 months

AAA

  1. 5-4.5 cm = yearly monitoring
  2. 5-5.5 = 3 monthly monitoring

> 5.5/ expands > 1cm a year = Endovascular annerysm repair.

How well did you know this?
1
Not at all
2
3
4
5
Perfectly
58
Q
A 68-year-old male visits his GP complaining of constipation, rectal bleeding and itchiness around his anus. He often feels ‘a lump’ hanging out after defecating which he has to push back in himself. On examination, anal tone is weak and a protruding mass is felt which has palpable muscular rings. What is the most likely diagnosis?
A Grade 3 haemorrhoids
B Grade 4 haemorrhoids
C Perianal abscess
D Type 1 rectal prolapse
E Type 2 rectal prolapse
A

E Type 2 rectal prolapse

A lump that has to be pushed back in on defecation = either a stage 3 hemorrhoid or rectal prolapse.

Haemorrhoid = an enlarged vascular cushion that has the tendency to protrude, bleed and prolapse.

Rectal prolapse = colapse of the rectum/part of the rectum out of the anus.

Type 1 = incomplete prolapse [just mucosa] - due to chronic straining, hemorrhoids, CONSTIPATION

Type 2 - complete prolapse[entire wall] hence palpable muscular rings - due to weaking of pelvic/anal musculature secondary to;
chronic STRAINING
Increasing age
NEUROLOGICAL DISORDER/SPHINCTER PARALYSIS

How well did you know this?
1
Not at all
2
3
4
5
Perfectly
59
Q
A 24-year-old female, who has recently returned from a 3-week trip to Vietnam, complains that she has been feeling ‘under the weather’ with fevers and joint pain. On direct questioning, she reveals that she had unprotected sexual intercourse with a stranger whilst in Vietnam. She is jaundiced and has right upper quadrant tenderness. Hepatitis B serology is requested. The results are shown below:
HBsAg +
HBeAg -
HBcAb IgM +
HBcAb IgG +
HBsAb -
What is the hepatitis status of this patient?
A Acute infection
B Chronic infection
C Cleared
D Vaccinated
E Susceptible
A

HBsAb - so hasn’t cleared the infection or been vaccinated as this needed to provide immunity.

Individuals that have been vaccinated against hepatitis B will only be HBsAb+. Previously infected patients who have cleared the virus will be HBsAb+ and HBcAb IgG+.

HBsAg + indicates infection
HBcAb IgM +
HBcAb IgG +
The earliest mark of acute infection will be a rise in HBsAg. This will be followed by a rise in HbcAb IgM and HBcAb IgG. HBcAb IgM will only be present during the acute phase of the infection.

Therefore this person has acute infection not a chronic infection.

If HBsAg is detected in the serum 6 months after an acute infection, it suggests that the patient has developed chronic hepatitis B. These patients will also have HBcAb IgG

How well did you know this?
1
Not at all
2
3
4
5
Perfectly
60
Q

A 61-year-old man is brought to A&E by his daughter as he has become increasingly breathless over the past 24 hours and he has been coughing up a large amount of green sputum. He has a past medical history of COPD. Arterial blood gases are requested which show the following results (on room air): pH : 7.33 (7.35-7.45) PaO2 : 6.7 kPa (> 10.6 kPa on air)
PaCO2 : 9.6 kPa (4.7 - 6 kPa on air) HCO3- : 33 mmol/L (22 – 28 mmol/L) Respiratory Rate : 22 /min

What is the diagnosis?
A Partially compensated respiratory acidosis B Fully compensated respiratory acidosis C Partially compensated metabolic acidosis D Fully compensated metabolic acidosis E Acute type 1 respiratory failure

A

A Partially compensated respiratory acidosis

pH = acidosis so not fully compensated

PaO2 = low 
PaCo2 = high [so resp acidosis]

HCO3- : High [partially compensated]

How well did you know this?
1
Not at all
2
3
4
5
Perfectly
61
Q
A 73-year-old man has come to the outpatient clinic with his wife. She says that her husband seems very confused on some days and then seems completely normal on others. During the consultation, the patient appears confused with an AMTS of 4/10. He is distressed and claims that he can see little men running across the desk towards him. The doctor also notices a resting tremor. What is the most likely diagnosis?
A Lewy body dementia
B Alzheimer’s disease
C Depressive pseudodementia
D Frontotemporal dementia
E Vascular dementia
A

A Lewy body dementia = fluctuation of confusion, HALLUCINATIONS + resting tremor [parksinsons like sx]

Accumulation of abnormal aggregates of proteins, called Lewy bodies in the cytoplasm of neurons. It also leads to a loss of dopaminergic neurons in the substantia nigra, resulting in features of parkinsonism (resting tremor, postural instability, bradykinesia and rigidity).

Frontotemporal dementia tends to first present with a change in personality or behaviour. Vascular dementia is caused by multiple small cerebral infarcts, leading to a loss of brain function. Patients may have a history of experiencing stroke-like symptoms. The patient’s state tends to undergo a step-wise decline in vascular dementia. Depressive pseudodementia is when dementia-like symptoms result from underlying depression. SBAs are likely to mention a recent bereavement or traumatic life event when alluding to depressive pseudodementia.

How well did you know this?
1
Not at all
2
3
4
5
Perfectly
62
Q
A 22-year-old teacher visits her GP after fainting several times over the past 2 months. She does not experience any palpitations, light-headedness or auras before she faints, and she recovers very quickly. She has not bitten her tongue or become incontinent at any point. When questioned about the timing of these episodes, she reveals that she has only ever collapsed at work after she has been writing on the whiteboard for quite some time. On examination, a firm, immobile lump is palpated in her left supraclavicular fossa. What is the most likely diagnosis?
A Paroxysmal atrial fibrillation
B Transient ischaemic attack
C Atonic seizures
D Subclavian steal syndrome
E Vasovagal syncope
A

Subclavian steal syndrome

Aetiology - compression [lump/extra rib] /stenosis [atheroscleorosis] proximal to vertebral artery.

Increased Q to arm = reduced Q to brain via vertebral artery [responsible for posterior circulation].

How well did you know this?
1
Not at all
2
3
4
5
Perfectly
63
Q
A 62-year-old diabetic on metformin sees his GP for a routine blood test. He claims that he has been compliant with his treatment and has not experienced any symptoms recently. His blood test reveals:
Na+ : 116 mmol/L (135-145)
K+ : 3.7 mmol/L (3.5-5)
Ca2+ : 2.4 mmol/L (2.2-2.6)
Total Cholesterol : 9.2 mmol/L (< 5)
Serum Albumin : 48 g/L (35 -50)
TFT - Normal
SST - Normal
What is the most likely cause of his hyponatraemia?
A Addison's disease
B Hypothyroidism
C Erroneous result
D Drug side-effect
E Nephrotic syndrome
A

Main findings:
Low sodium
High Cholesterol

Moderate hyponatremia [>125] = Headaches/nause+V/cramps [think about retired hurt cricket game]

Severe {<120] = [Neuro Sx- seizures, hallucinations, confusion, memory loss]

Pt is assymptomatic so must be an erronous result.

How well did you know this?
1
Not at all
2
3
4
5
Perfectly
64
Q

Which of the following lung pathologies produces the ‘sail sign’ appearance on CXR?
A Right upper lobe collapse B Right middle lobe collapse C Right lower lobe collapse D Left upper lobe collapse
E Left lower lobe collapse

A

Left lower lobe collapse - collapsed lobe = second heart border

heart shadow + 2nd heart border = sail sign appearnace.

65
Q
A 62-year-old man presents with severe, acute epigastric pain with nausea and vomiting. The pain radiates to the back and improves when sitting forward. It started 4 days ago, but the patient assumed it was indigestion and refused to come to hospital. On examination, there is epigastric tenderness and ecchymoses over the periumbilical area and flank. The patient drinks in moderation and has not had any alcohol recently. Serum amylase is 600 U/L (< 140). Which investigation should be performed to confirm the diagnosis?
A ERCP
B Abdominal CT scan
C Abdominal X-ray
D Abdominal ultrasound
E MRCP
A

Dx - Pancreatitis

1st line dx = US
2nd line = CT for staging + complications

66
Q

What is Cushings reflex?

Becks Triad?

A

Raised ICP

HTN
Irregular breathing
Bradycardia

Tamponade physiology
Raised JVP
Muffled heart sounds
Hypotension

67
Q
A 61-year-old male comes to A&amp;E complaining of chest pain and mentions that he can feel his heart ‘pumping out of his chest’. An ECG shows regular broad complex tachycardia, with no P waves. His blood pressure is 124/87 mm Hg. How should this patient be treated?
A Defibrillation
B DC cardioversion
C Amiodarone
D Adenosine
E Atropine
A

Amiodarone

Dx - VT

Pulseless VT and VF = defib

Hemodynamically stable VT = IV amiodarone otherwise defib.

68
Q
72-year-old man has recently suffered a stroke. He has recovered well and appears to have regained much of his physical strength, however, his speech has changed quite considerably. His daughter says that he will talk the same amount as he always did but his sentences will not make any sense, and he doesn’t seem to notice. When asked to describe what he did this morning, he responds: ‘the bugle fidget and that I played tractor to you before’. Damage to which part of the brain is likely to manifest in this way? A Wernicke’s area 
B Broca’s area 
C Arcuate fasciculus 
D Hippocampus 
E Amygdala
A

Wernicke’s [understanding of speech]
Lesion here - individual can speak but can’t understand what they are saying - so speech is non-sensical.
[temporal lobe]

Broca’s area - Aphasia
[for speach]
[frontal lobe]

69
Q

An inpatient on the surgical ward is recovering after having a kidney stone removed. A routine blood test is performed which shows the following results:
Na+ : 135 mmol/L (135 – 145)
K+ : 8.7 mmol/L (3.5 – 6.0)
Ca2+ : 0.2 mmol/L (2.2 – 2.6)
An ECG is performed which shows no obvious abnormalities.
Given the above information, what should be the next step in the management of this patient?
A Urgently draw another blood sample
B 10 mL 10% calcium gluconate
C 20 mL 20% calcium gluconate
D 50 mL 50% dextrose + 10 U insulin
E IV salbutamol

A

Urgently draw another blood sample

Red cell lysis due to blood drawing error = release of intracellular K+ and low Ca2+ indicates this.

70
Q
A 59-year-old man is brought to A&amp;E by his daughter after having collapsed at home. He has very little recollection of the incident – one minute he was doing the dishes, and next minute he was lying on his back on the floor. He has no history of recent head trauma and mentions that he felt ‘absolutely fine’ when he regained consciousness. An ECG is performed showing bradycardia (36 bpm) and dissociation between the p waves and QRS complexes. A diagnosis of complete heart block is made. What might be seen on close inspection of his JVP?
A Large V waves
B Cannon A waves
C Kussmaul sign
D Raised JVP with absent pulsation
E Slow Y descent
A

Cannon A waves - in complete heart block atria and ventricular contraction is completely dissociated [so if atria and ventricules contract simultaneously = cannon A wave].

Large V waves - increased venous return [seen in tricuspid regurg]

Kussmaul sign - impaired right ventricular filled [restrictive cardiomyopathy/constrictive pericarditis]

Raised JVP with absent pulsation - SVC obstruction

Slow Y descent - impaired right ventricular filling. [e.g tricuspid stenosis]
Y = flow of blood from the right atrium into the right ventricle through an open tricuspid valve

71
Q

Which of the following options fits the criteria for giving long-term oxygen therapy in COPD?
A PaO2 7.3-10 kPa despite maximal treatment
B PaO2 7.3-10 kPa and pulmonary hypertension
C PaO2 < 7.3 kPa despite maximal treatment
D PaCO2 > 6 kPa despite maximal treatment
E PaCO2 > 8 kPa despite maximal treatment

A

PaO2 < 7.3 kPa despite maximal treatment

or 7.3-8kpa w/
Pulm Hypert
Secondary Polycythaemia 
PULM OEDEMA 
NOCTURNAL HYPOXIA
72
Q
A 71-year-old female, with a history of atrial fibrillation, presents to A&amp;E with severe, diffuse abdominal pain. Her blood pressure is 84/60 mm Hg and her pulse is irregularly irregular with a rate of 130 bpm. Abdominal examination is normal. An abdominal X-ray is performed. Which of these radiological features is most likely to be seen?
A Rigler’s sign
B Pneumoperitoneum
C Toxic megacolon
D Gasless abdomen
E Coffee bean sign
A

Gasless abdomen

AF = Rf for ischaemic bowel disease 
others = pro-coagulant state and hypotension. 

Ischaemic bowel disease includes:

  1. )Acute mesenteric ischaemia
  2. ) chronic mesenteric ischaemia

Acute mesenteric ischaemia = triad [severe abdo pain, normal exam + hypotension]

X-ray = gaseless abdo
Thickened bowel wall
Pneumatosis = air in bowel wall secondary to necrosis

Chronic mesenteric ischaemia = [↓ flow = gut claudication ∴ colicky post-prandial abdo pain

73
Q

Which of the following is not part of the criteria for diagnosing sepsis?
A Heart Rate > 90 bpm
B Respiratory Rate > 20 breaths per minute
C Temperature > 38°C
D White Cell Count < 4 x 109/L
E Blood Pressure < 90/60 mm Hg

A

Blood Pressure < 90/60 mm Hg

Sepsis criteria

HR>90bpm
RR >20/PCO2 <4.3kpa
White cell <4x10^9/L or >12x10^9/L
Temperature > 38°C

Sepsis 6 
3 in 
Abx 
O2 
Fluids 

3 out
Lactate
Urine
Blood cultures

74
Q

A 47-year-old man presents to his GP having experienced a few episodes of haemoptysis over the past month. He returned from a holiday to Bangladesh 6 weeks ago. On direct questioning, he admits to losing approximately 5 kg in weight over the past month and he has had to replace his bed sheets more frequently because they are often drenched with sweat when he wakes up in the morning. A chest X-ray reveals an area of consolidation in the right upper zone. Sputum microscopy using Ziehl-Neelsen stain reveals acid-fast bacilli. What is the most appropriate treatment option?
A Rifampicin and isoniazid for 6 months; ethambutol and pyrazinamide for the first 2 months
B Ethambutol and pyrazinamide for 6 months; rifampicin and isoniazid for the first 2 months
C Rifampicin and isoniazid for 6 months; ethambutol and pyrazinamide for the first 4 months
D Rifampicin and pyrazinamide for 6 months; ethambutol and isoniazid for the first 2 months
E Rifampicin, pyrazinamide, ethambutol and isoniazid for 6 months

A

Rifampicin and isoniazid for 6 months; ethambutol and pyrazinamide for the first 2 months

RIPE mneumonic

R and I
rifampicin and isoniazid for the first 6 months

P and E
Ethambutol and pyrazinamide for first 2 months

All drugs are used in the

All four drugs are started at the same time Rifampicin and Isoniazid – 6 months Pyrazinamide and Ethambutol – 2 months These drugs can be remembered using the mnemonic: RIPE N.B.

Pyridoxine (vitamin B6) is given alongside TB treatment because isoniazid leads to vitamin B6 deficiency, which causes peripheral neuropathy

75
Q

A 28-year-old man has experienced several episodes of sweating, palpitations and anxiety over the past 6 months. He has a past medical history of thyroid cancer (aged 19) which was treated with total thyroidectomy. What is the most appropriate investigation?
A Serum 17-hydroxyprogesterone levels
B 24 hr urine 5-hydroxyindoleacetic acid levels
C 24 hr urine vanillylmendelic acid
D Plasma adrenaline concentration
E Thyroid uptake scan

A

Phaechromacytoma = tumour of adrenal medulla = catecholamines

Catecholamines are metabloised + it’s metabolites can be measured in the urine:

  • 24 hr urine vanillylmendelic acid
  • Metanephrine
Men 2A syndrome [Multiple endocrine neoplasia] 
Triad:
Thyroid cancer 
Parathyroid adenoma 
Phaechromocytoma
76
Q
A 76-year-old woman is brought to A&amp;E by her daughter. She is complaining of severe left iliac fossa pain accompanied by nausea, vomiting and fever. On inspection, she shows signs of peritonism. Vital signs: HR = 123 bpm, RR = 24 bpm, Temp = 38.7°C and BP = 87/54 mm Hg. An erect CXR is requested, which shows air under the diaphragm. A diagnosis of perforated diverticulitis localised to the sigmoid colon is made. What is the most appropriate surgical procedure?
A Left colectomy
B Abdominoperineal resection
C Hartmann’s procedure
D Delorme procedure
E Anterior resection
A

Removal of sigmoid colon:

  1. ) Primary anastamosis [won’t heal in this pt so contraindicated]
  2. ) Hartmann’s procedure {end colonostomy w/rectal stump - anastomosis can be done at a later date].

Delorme procedure - full thickness rectal prolapse.

Left colectomy - used in colorectal cancer of descending colon
[removal colon 2/3 way along transverse colon to start of sigmoid colon]

Abdominoperineal resection [rectal tumours in the bottom 1/3 of rectum]

Anterior resection [rectal tumours in top 2/3 of rectum]

77
Q
An inpatient on the respiratory ward is currently undergoing treatment for a pneumonia that he developed 2 days ago. A blood test is performed which shows a low white cell count, with a particularly low neutrophil count. The patient is re-examined and found to have a considerably enlarged spleen. On further questioning, He has suffered from 3 infections in the past 5 months and complains that his rheumatoid arthritis has been getting worse.
What is the most likely diagnosis?
A Malaria
B Tuberculosis
C Felty’s syndrome
D Lymphoma
E Chronic lymphocytic leukaemia
A

Felty’s syndrome - RhA + neutropenia+ Splenomegaly

SANTA 
SPENOMEGALY 
ANAEMIA 
NEUTROPINA 
THROMBOYCTOPAENIA
ARTHRITIS
78
Q
What is a major consequence of raising plasma sodium concentration too rapidly?
A Stroke
B Rhabdomyolysis
C Central pontine myelinolysis
D AKI
E Cerebral oedema
A

Central pontine myelinolysis: [damage to myelin sheath of pons]

Acute paralysis, dysarthria and dysphagia.

When the correction is too rapid, the brain cells do not have enough time to adapt to the increasing extracellular osmolality and the osmotic gradient between the myelin cells and the extracellular environment will draw water out of the myelin cells, ultimately resulting in CPM.

79
Q

[3]

A

Start intravenous K+ supplement 40 mmol/L in the first bag of 0.9%
normal saline - is incorrect as first line treatment of DKA is with IV saline fluid replacement - K+ replacement not at this stage.

DKA = Hyperglycaemia, increased blood ketones and metabolic acidosis

Signs DKA - N+V/ polyuria/dypsia/ Kaussmaul breathing [compensate for met acisosis]

  • Initial treatment of DKA aims at correcting severe volume depletion, again with IV saline infusion at a rate of 1-1.5L for the first hour. When glucose reaches 11.1mmol, fluid should be changed to 5% dextrose to prevent hypoglycaemia.

Treat DKA

  1. IV insulin with saline
  2. K+ as insulin drives K+ into cells

Bicarbonate therapy may be necessary to help correct acidosis.

80
Q

[7] A 70-year-old woman has recently been admitted to hospital with a chest
infection and mild confusion. Unfortunately no informant was available,
hence a full history could not be taken. You are called to see her 2 days after
her admission because she was sleeping poorly, complaining of nausea and
sweating, and went on to have a seizure.
Which of the following is the most likely reason for this clinical picture?
A Alcohol withdrawal
B Opiate use
C Opiate withdrawal
D Sedative use
E Sedative withdrawal

A

Symptoms of sedative withdrawal:

  • Insomnia, seizures + delirium
  • Increased ANS Sx - e.g sweating
  • N+V
81
Q
Which of the following drugs is used during cardiac arrest to improve the
coronary perfusion pressure?
A Adrenaline
B Amioderone
C Atropine
D Calcium gluconate
E Magnesium sulphate
A

Adrenaline

ALS guidelines - pulseless VT/VF

  1. Defib
  2. CPR + asses rhythmn
  3. Give adrenaline every 3-5 mins

Amioderone after 3 shocks

Assystole/PEA i.e non-shockable rhythmn

  1. CPR
  2. Adrenaline every 2-3 mins
82
Q
[Q23] Which of the following conditions do not cause a leucoerythroblastic
picture on blood film?
A Chronic myeloid leukaemia
B Megaloblastic anaemia
C Metastatic spread to the bone marrow
D Myeloma
E Tuberculosis
A

Megaloblastic anaemia = no bone marrow invasion.

leucoerythroblastic CHANGE = presence of immature cells due to bone marrow invasion:

  • Infective - TB
  • Malignant - CML/myeloma

-

83
Q

A new tumour marker is being tested for use in cholangiocarcinomas. In a
trial 40 patients have been tested. The trial produces 10 positive results and
30 negative results. Of the 10 positive results, 5 of them are false positives.
Of the 30 negative results, 5 of them are false negatives.
What is the specificity of this test?
A 17%
B 50%
C 70%
D 83%
E 100%

A

Specificity = low false negative

∴ true negative rate = specificity= 25/30 = 83%.

Sensitivity = low false possitive

84
Q

A 43-year-old woman presents with a 1-week history of loin pain,
suprapubic pain and dysuria. On examination she looks unwell, has a pulse
of 110 bpm, blood pressure of 120/80 mmHg, respiratory rate of 24/min
and a temperature of 38.5°C. Bloods are taken for routine tests and blood
cultures, a catheter passed, and empirical antibiotics started. Later that day,
blood cultures come back showing a Gram-negative rod. You note that in
the last 3 hours she has only passed 30 ml of urine and she is starting to
show signs of confusion. Observations remain as they were earlier.
Which of the following terms best describes the patient’s physiological
status?
A Sepsis
B Septic shock
C Septicaemia
D Severe sepsis
E Systemic inflammatory response syndrome

A

≥2 of this criteria = severe sepsis.

SIRS + Septicaemia = sepsis

Sepsis Dx criteria: 
HR >90
RR>20/ PCO2 <4.3kpa 
WCC: <4x10^9/>12x10^9 
Fever 

≥2 of this criteria = severe sepsis.

septic patient is showing signs of organ
hypoperfusion – for instance, oliguria (kidney hypoperfusion), confusion
(brain hypoperfusion) or serum lactate >4 (muscle hypoperfusion). This
woman is oliguric and confused. Shock means hypotension, and septic
shock is defined as sepsis with refractory hypotension (i.e. hypotension
persevering in spite of fluid resuscitation), which does not apply to this
patient, therefore she is best described as having severe sepsis. This may
indicate that present antibiotic therapy is inadequate and that aggressive
intravenous fluid resuscitation should be instituted.

85
Q

[19]A 21-year-old woman is admitted to the emergency department with
increasing shortness of breath after taking oral amoxicillin, which
was started by her GP for a chest infection. Chest auscultation reveals
widespread wheeze bilaterally. Vascular access is secured.
Which of the following management options is NOT appropriate?
A Discontinue administration of the suspected drug
B Give intramuscular adrenaline immediately
C Give intravenous 5% dextrose immediately
D Give intravenous chlorphenamine
E Give intravenous hydrocortisone

A

Give intravenous 5% dextrose immediately
using 5% dextrose for fluid resuscitation as it
quickly redistributes itself out of the intravascular space.

Treatmement of anaphylaxis 
SECURE ABC 
1. IM adrenaline 
2. Anti-histamines [chloramphenamine] 
3. intravenous hydrocortisone [reduces late phase Sx]
86
Q

[20]A 29-year-old man with known type 2 diabetes mellitus and excess
alcohol intake was admitted to the emergency department with reduced
consciousness. He is on oral anti-diabetic agents (intermediate and longacting
sulphonylureas) for his diabetes. His friend who brought him said
that he had been drinking excessive quantities of alcohol that evening.
Which of the following managements is NOT appropriate in this case?

A Consider CT of the head
B Continue intravenous dextrose after the blood glucose level has returned
to within normal range
C Intravenous dextrose
D Intramuscular glucagon
E Send random blood glucose sample to the laboratory

A

Intramuscular glucagon may not work in alcohol-related hypoglycaemia,
liver disease or prolonged hypoglycaemia.

87
Q

[40] A 46-year-old woman presents to the emergency department in a reduced
state of consciousness. Her blood glucose level was 9 mmol/L. On
examination, her body temperature was 33°C. She has non-pitting oedema
on the skin of her hands, feet and eyelids. No obvious head injury is noticed.
She has a thyroidectomy scar on her neck.
What is the most likely diagnosis?

A

MYXOEDEMA COMA

  • LOC
  • HYPOTHERMIA
  • CONVULSIONS
88
Q

HYPOCALCAEMIA SX?

A

CATS GO NUMB

CONVULSIONS
ARRHYTHMIA
TETANY
NUMBNESS/PARASTHESIA

89
Q
Which of the following is not a sign of cirrhosis?
A Gynaecomastia
B Asterixis
C Koilonychia
D Hepatic fetor
E Clubbing
A

Koilonychia - spoon shaped nails [chronic iron deficiency anaemia]

Note Hepatic fetor = bad breath due to reduced live detoxification.

90
Q
A 24-year-old swimwear model presents to A&amp;E with severe right iliac fossa pain. The pain was initially poorly localised to the umbilical region, before moving to the right iliac fossa. This has been accompanied by nausea, anorexia and fever. A diagnosis of appendicitis is made and she is referred for an appendicectomy. What is the most suitable surgical incision for this patient?
A Lanz 
B Kocher 
C Pfannenstiel 
D Rutherford-Morrison 
E Gridiron
A

A - LANZ

2 types of appendicectomy scars:

  1. LANZ along bikini line
    - advantage: Cosmetic appearance
    - Disadvantage: Increased risk of herniation.
  2. Gridiron - Incsion made at mcburneys point [1/3 the distance laterally from ASIS to umbilicus] - runs perpendicular to line between ASIS and umbilicus.

Kocher
Right Kocher – Open Cholecystectomy
Left Kocher – Splenectomy

Pfannenstiel – Caesarian Section, Gynaecological Operations

Rutherford-Morrison [hockey stick scar] - kidney transplant.

91
Q

A 55-year-old man presents to his GP having coughed up blood on several occasions over the past 6 months. He said that he has had a ‘smokers’ cough’ for years but the appearance of blood has been a recent change. On direct questioning, he admits to unintentionally losing about 5 kg of weight over the past 6 months. A chest X-ray shows a 2 cm cavitating lesion in the right upper lobe. There appears to be a few other smaller nodules surrounding the large cavitating lesion. The left lung appears slightly fibrosed, but is otherwise normal. What is the most likely diagnosis?
A Small cell lung cancer
B Squamous cell lung cancer C Atypical pneumonia
D Lung abscess
E Goodpasture’s syndrome

A

Squamous cell lung cancer

  • Small cell carcinoma = the highest incidence in smokers, but this question refers to the fact that tumour is associated with a cavitating lesion so answer is squamous cell carcinoma.
92
Q
A 32-year-old man, with a history of IV drug abuse, presents to A&amp;E with a high fever and rigors. He has also been very breathless and has experienced epigastric pain that is worse on exertion. On examination, giant V waves are seen in the JVP and tender, pulsatile hepatomegaly is palpated. What is the most likely diagnosis?
A Mitral stenosis
B Tricuspid regurgitation
C Pulmonary hypertension
D Portal hypertension
E Viral hepatitis
A

B = Tricuspid regurgitation secondary to infective endocarditis.

Epigastric pain that is worse on exertion - classically tricuspid regurgitation.

Ventricular contraction pushes a column of blood through the valve and right atrium up the superior vena cava, appearing as ‘giant V waves’ in the JVP.

The backflow of blood from the right side of the heart down the inferior vena cava, due to tricuspid regurgitation, leads to hepatic venous congestion and tender, pulsatile hepatomegaly.

Other features of tricuspid regurgitation include pansystolic murmur heard loudest at the lower left sternal edge during inspiration, parasternal heave, ascites and ankle oedema

Note Tricuspid regurg causes RHF.

Causes of RHF:

  1. LHF
  2. Lungs - Pulm HyperT
  3. Valvular [tricuspid regurg/pulmonary valvular disorder].
  • V waves = Increased venous return in late systole- so Large V waves are seen in tricuspid regurgitation.
  • A waves [3rd heart block]
  • Y waves = flow of blood through open tricuspid valve

Raised JVP with absent pulsation occurs in superior vena cava obstruction.

Slow Y descent is associated with tricuspid stenosis.

93
Q

Acanthosis nigricans what is it and what conditions is it found in?

A

Acanthosis nigricans - area of dark velvety skin usually @ body folds - secondary to endocrine disorders [cushings/T2DM/PCOS] + GI malignancy.

94
Q

Drugs that cause gynocomastiae?

A

Spironolactone
Cimetidine
Oestrogens
Digoxin

95
Q
Which part of the prostate gland undergoes progressive hyperplasia in BPH?
A Central zone
B Transitional zone
C Peripheral zone
D Ejaculatory duct
E Anterior fibromuscular stroma
A

BPH - nodular hyperplasia of the trasitional zone which surrounds the urethra as it enters the prostate gland - hence urinary symptoms.

Peripheral zone = glandular tissue is more commonly affected in prostate cancer.
[peripheral zone but tranbsitional cell carcinoma].

96
Q

Lung conditions that cause cavitating lesions [has an air-fluid level]?

A

Lung conditions that cause cavitating lesions [has an air-fluid level];

Pneumonia:

  • Staph aureus
  • Klebsiella

TB

Lung cancer:
- Squamous cell carcinoma

Inflammtory:

  • RhA
  • Wegners
97
Q
[25] 
A 77-year-old man is referred to the outpatient clinic by his GP having presented with chest pain and worsening shortness of breath. He has a history of COPD, diagnosed 12 years ago. On examination, his JVP is elevated, a parasternal heave is palpated and auscultation reveals an early diastolic murmur. An ECG is performed which shows right-axis deviation, a tall R wave in V1 and peaked P waves in lead ll. What is the most likely diagnosis?
A Aortic regurgitation
B Mitral stenosis
C Pulmonary hypertension
D Right heart failure
E Exacerbation of COPD
A

COPD = Chronic hypoxia = pulm vasoconstriction = Pulm hyperT

On examination, his JVP is elevated, a parasternal heave is palpated and auscultation reveals an early diastolic murmur. = aortic/pulmonary regurgitation. Aortic regurgitation is unlikely as the examination and ECG findings are consistent with a right-sided heart problem. This murmur is therefore due to pulmonary regurgitation (because of pulmonary hypertension) and is known as a Graham Steell murmur. + RV hypertrophy.

An ECG is performed which shows right-axis deviation, a tall R wave in V1 and peaked P waves in lead ll.

= Peaked P-waves in lead II = p.pulmonale + right-axis deviation, a tall R wave in V1
= right ventricular hypertrophy.

∴ more evidence for pulm hypertension than rHF
- RHF = more signs e.g portal hypertension, ankle oedema e.t.c

98
Q
A 54-year-old female, with a BMI of 28, presents with a 2-year history of epigastric pain that radiates to the neck. It gets worse when lying down, and she also complains of painless regurgitation of food. What is the most appropriate investigation to confirm the diagnosis?
A Chest X-ray
B Barium swallow
C ECG
D OGD
E Manometry
A

Painless regurgitation of food + GORD symptoms = Hiatus hernia [occurs due to increased intatoric pressure - causing displacement of stomach through oesophageal hiatus].

Ix - Barium Swallow.

There are two types: Sliding (gastro-oesophageal junction (GEJ) slides into the thorax), and Rolling (GEJ remains in place but a bulge of fundus herniates into the chest alongside the oesophagus). Around 80% of cases are sliding hiatus hernias.

99
Q
A 41-year-old female is referred to the dermatology clinic because she has developed multiple purple nodules on her shins. They are tender and have a diameter of 1-2 inches. Erythema nodosum is suspected. Which of the following is not a cause of erythema nodosum?
A Tuberculosis
B Reaction to sulphonamides
C Inflammatory bowel disease
D Ankylosing spondylitis
E Behçet’s disease
A

Erythema nodusum = Panniculitis violaceous nodules on skin - esp shins

Causes = LOST BUSH

L - leprosy 
O - OCP + preganacy 
T- TB 
S - sarcoidosis/sulphonamides 
T - TB 

B- Behcet’s
U- UC/Chron’s
S - Salmonella + strep [to SS - think last two letters of CHESS]
H- Histoplasmosis

100
Q

A 44-year-old female presents to her GP complaining of worsening hearing. A full cranial nerves examination is performed. When Weber’s test is performed, she hears the sound louder in her left ear. Then, Rinne’s test is performed and she reports that, in both ears, the sound is loudest when the tuning fork is held in front of the auditory canal rather than when the fork is held against her mastoid processes. Which of the following best describes the patient’s condition?
A Conductive hearing loss in the right ear
B Conductive hearing loss in the left ear
C Sensorineural hearing loss in the right ear
D Sensorineural hearing loss in the left ear
E Bilateral sensorineural hearing loss

A

Sensorineural hearing loss in the right ear.

Louder infront of ear canal and not mastoid porocess - indicates sensorineural hearing loss.

Sensorineural hearing loss - the sound is loudest in the good ear.
- Weber’s test is performed, she hears the sound louder in her left ear.

So deficit is in the right ear and it’s sensorineural.

101
Q

What test is used to identify SVC obstruction?

A

Pemberton’s test:

  • raise arms above your head for one minute:
  • Facial plethora
  • Raised non-pulsatile JVP
  • Inspiratory stridor

Features of SVCS are dyspnea, orthopnoea, swollen face and arms, plethora, cough and engorged neck and facial veins (raised and non-pulsatile JVP)

102
Q
27. A 28-year-old IV drug-user visits his GP practice for an annual check-up. Hepatitis serology is requested and the following results are reported.
HBsAg -
HBeAg -
HBcAb IgM -
HBcAb IgG +
HBsAb +
What is the hepatitis status of this patient?
A Acute infection
B Chronic infection
C Cleared
D Vaccinated
E Susceptible
A

HBsAg - so does not have an acute infection.
HBsAb + - has immunity so either vaccinated or infection has been cleared.

HBcAb IgM -
HBcAb IgG +
Therefore infection has been cleared as antibodies to Hbc present.

Note if chronic infection
HbsAg+
HbcAb IgG+ [chronic]
HBsAb +

HBcAb IgM will only be present during the acute phase of the infection. HBcAb IgG will persist once the virus is cleared or if the infection becomes chronic.

103
Q
A 24-year-old female is brought to A&amp;E having fallen off a stage in a nightclub whilst under the influence of LSD. Her eyes open when the registrar squeezes her trapezius and she makes a few incomprehensible sounds. Her arms flex, wrists clench and legs extend and internally rotate in response to pain. What is her GCS?
A 5
B 7
C 8
D 9
E 10
A

E - 2
v - 2
M- 3

B = 7

“Her arms flex}

V - [1-5]
1 - no sound 
2 - few sounds
3 - Few words 
4 - Confused 
5 - orientated in time and space 
M [1-6]
1 - no movement 
2- Abnormal extension 
3- Abnormal Flexion
4-Localises pain 
5- Withdraws from pain 
6- Obeys comand
104
Q

[10]A 34-year-old man is brought to A&E having collapsed in a shopping mall. He did not lose consciousness but mentioned that he felt dizzy and could feel his ‘heart racing’. He has a past medical history of asthma. His ECG shows a regular narrow
complex tachycardia with no visible P waves. Vagal manoeuvres failed to terminate the tachycardia. What is the next most appropriate step in the management of this patient?
A Verapamil
B Amiodarone
C Adenosine
D Bisoprolol
E Flecainide

A

SVT management:

  1. Vagal manoeveres
  2. IV adenosine [6mg, 12mg, 12mg]

If stable

  1. Verapamil/beta-blocker
  2. DC cardioversion

If unstable
1. DC cardioversion

In this pt IV adenosine is contraindicated because he is athmatic = bronchospasm.

So answer = verapamil.

105
Q
[11] Which of the following is a cause of primary amenorrhoea? 
A Prolactinoma 
B Pregnancy 
C Haemochromatosis 
D Polycystic ovarian syndrome 
E Turner syndrome
A

Turner syndrome - genetic [45X] - lacking an X chromosome.

  • Short stature
  • Cubitus Valgus (where the forearm is angled away from the body to a greater degree than normal when fully extended)
  • Gonadal dysgenesis (defective development)
106
Q
[12] In which part of the nephron does bendroflumethiazide have its effect?
A Proximal convoluted tubule
B Descending limb of the loop of Henle
C Ascending limb of the loop of Henle
D Distal convoluted tubule
E Collecting duct
A

Proximal convoluted tubule - Carbonic anhydrase inhibitors - increase bicarbonate excretion, which, in turn, increases sodium excretion via Na+/Bicarb co-transporter.

Descending limb of the loop of Henle - OSMOTIC DIURETICS [reduce gradient - freely filtered but poorly absorbed]

Ascending limb of the loop of Henle [loop diuretics - inhibit Na+/K+/Cl- tripple transporter]

Distal convoluted tubule [thiazide diuretics inhibit Na+/Cl- transporter ]

Collecting duct - K+ sparing - inhibit aldosterone mediated Na+ absorption [spironolactone]

107
Q
Which of the following most accurately describes the sodium and potassium requirements of a 70 kg man over a 24 hour period?
A 100 mmol Na+ &amp; 40-50 mmol K+
B 120 mmol Na+ &amp; 5-10 mmol K+
C 120 mmol Na+ &amp; 10-20 mmol K+
D 120 mmol Na+ &amp; 60-70 mmol K+
E 140 mmol Na+ &amp; 60-70 mmol K+
A

70kg man requires

140 mmol Na+
60-70 mmol K+

3L of water

Note 1L of saline has about 155mmol Na+

108
Q
70-year-old man presents with a 3-month history of polyuria. He has been urinating up to 12 times per day and has also experienced some constipation, abdominal pain and back pain. More recently, he has noticed that his face appears ‘puffier’ than usual and his ankles are swollen. He is on citalopram to treat his depression and takes no other regular medications. Blood tests reveal: ESR = 64 mm/hr (< 22). Urinalysis reveals:
Protein: Positive
Blood: Negative
24 hr urine protein (g): 9.8 (< 3.5)
What is the most likely diagnosis?
A Cushing’s syndrome
B Amyloidosis
C Glomerulonephritis
D Malignancy
E Congestive cardiac failure
A

Amyloidosis secondary to multiple myeloma.

polyuria
Constipation 
abdominal pain
back pain
Depressive medication 

Stones, throans, abdo moans, psychic groans - sx of hypercalcaemia.

face appears ‘puffier’ than usual and his ankles are swollen
Raised ESR
urine Protein: Positive
[renal sx]

Hypercalcaemia + bach ache + renal failure

[CRAB] - multiple myeloma
+ESR is raised

Multiple myeloma not an option - but Ig light chains from multiple myeloma can predispose to AL - amyloid fibrils.

Amyloidosis

109
Q
A 36-year-old man presents with a 3-week history of fatigue, frequent urination and excessive thirst. He also mentions that he has been unable to take part in his weekly 5-a-side football sessions for the past month because his ‘muscles feel weak’. The patient’s notes reveal that, during a previous appointment 6 months ago, his blood pressure was measured at 164/98 mm Hg. He was offered a follow-up appointment to discuss management options, however, he did not attend. The GP measures the patient’s blood pressure again, and it is 172/102 mm Hg. What would you expect to see on the ECG of this patient?
A Tented T waves
B Absent P waves
C ST elevation
D J waves
E U waves
A

U waves - hypokalemia secondary to CONNs syndrome

CONNs - hyperaldosteronism = high BP + increasing Na+ reabsoprtion and K+ excretion.

Hypokalaemia induces nephrogenic diabetes insipidus, which, consequently, leads to polyuria and polydipsia

ECG indicators of hypokalaemia?

  • Flattened T-waves
  • U waves
  • ST depression
  • prolongued PR

Hyperkalemia;

  • Tented T-waves
  • Prolongued PR
  • Flattened P-waves
  • Prolongued QRS
110
Q
A 40-year-old man is brought into A&amp;E after he was found lying unconscious on the side of the road with an empty bottle of whisky next to him. Once he regains consciousness, he starts yelling at the ward staff expressing that he thinks he has been kidnapped and is being held hostage. He jumps out of bed, but finds it difficult to walk. He has a wide-based gait and he is taking small steps. Given the most likely diagnosis, what should form part of the immediate management?
A Acamprosate
B Chlorediazepoxide
C Thiamine
D Naloxone
E Disulfiram
A

Alcoholic + triad of:

  • Ophthalmoplegia
  • Ataxia
  • Mental state change

= Wernicke’s encephalopathy

tx = urgent IV pabrinex [thiamaine replacement] to prevent permanent development Korsakoff psychosis.

  • amnesia [retro and anterograde]
  • Confabulations [making up shit to fill blanks in memory].
111
Q
Which of the following is not a reversible cause of cardiac arrest?
A Hypothermia
B Tension pneumothorax
C Cardiac tamponade
D Hypokalaemia
E Pleurisy
A
4H's and 4T's 
Hypoxia
Hypothermia 
Hypovolaemia
Hypo/hyperkaleamia 

THROMBOEMBOLIC
TENSION PNEUMOTHORAX
Toxins
Tamponade

112
Q
An 82-year-old lady has been in hospital for 4 weeks due to a hip fracture that she sustained after falling at home. Due to her immobility, a pressure sore has developed on the heel of her right foot. There is an intact fluid-filled blister measuring 3 inches in diameter. The ulcer is superficial and there is no subcutaneous tissue visible. According to the EPUAP, what grade of severity is this pressure ulcer?
A Grade 1
B Grade 2
C Grade 3
D Grade 4
E Ungradable
A

GRADE 1 - intact skin
Grade 2 - Blisters/open wound
Grade 3 - Wound involves deep layers of skin [subcut tissue]
Grade 4 - Visible bone/muscle/tendon

113
Q
16. A 43-year-old woman presents with a ‘rather embarrassing’ problem. Since the birth of her fourth child, 3 months ago, she has wet herself several times. She has noticed that whenever she laughs or coughs, a little bit of urine leaks out without her control. What is the name of this type of incontinence?
A Functional incontinence
B Stress incontinence
C Urge incontinence
D Overflow incontinence
E Double incontinence
A

Stress incontinence - due to increased intrabdo pressure
[incompetent sphincter]

Functional incontinence - patients are unable to find a toilet before urinating e.g stuck in wheelchair.

Urge incontinence - soon as urge = piss ur pants [ DETRUSSER OVERACTIVITY (due to a neurological problem or an intrinsic problem with the detrusor muscle)

Double incontinence = piss and shit [neuro disorders - parkinsons]

Overflow incontinence is the involuntary release of urine from a full bladder.

114
Q
21. A 56-year-old man comes to A&amp;E with a very swollen glans. He went to the toilet to urinate last night, however, once he had finished, he was unable to replace his foreskin back over his glans. Since then, his glans has gradually become very painful and inflamed. What is the name of this condition?
A Phimosis
B Paraphimosis
C Balanitis
D Priapism
E Peyronie’s disease
A

Paraphimosis - tight foreskin is retracted and then cannot be replaced back over the glans.

Phimosis is when a tight foreskin occludes the meatus.

Balanitis - inflammation of gland [reactive arthritis - reiters syndrome]

115
Q
29. Which of the following is not a risk factor for the formation of a DVT?
A Factor V Leiden
B Malignancy
C Nephrotic syndrome
D Antiphospholipid syndrome
E Alcohol
A

Answer = Alcohol

DVT RF - Virchows triad:

Stasis

Hyperocagulanility
- nephrotic syndrome,
[increased excretion of antithrombotic factors by the affected kidneys and increased production of pro-thrombotic factors by the liver]

Endothelial damage

116
Q

What murmur has a very loud first heart sound.?

A

Mitral stenosis - loud S1 [opening snap]

Other features

  • Mid- diastolic murmur
  • Opening snap
  • Ass w/rheumatic fever
  • Malar flush
117
Q

A tapping apex beat suggests what diagnosis?

A

Tapping apex beat = a palpable [hence loud] 1st heart sound from a stenosed mitral valve.

The sound is louder because - in mitral stenosis the blood from the atria hasn’t fully emptied into the ventricles by the start of systole.
So valve leaflets are open and need to be slammed shut for systole = loud therefore palpable 1st heart sound.

118
Q

Whats the difference between mitral and tricuspid regurg?

A

Mitral is louder - left ventricular = higher pressure

So tricuspid regurg = softer pan-sysolic murmur.

119
Q
  1. A 50 year old complains of slowly worsening weakness in his left leg. This started gradually three months ago, and is now making him limp. Examination reveals increased tone, brisk reflexes and weakness in the left leg. - whats DX?
A

You need to recognise these as upper motor neurone signs. It is slowly progressive, so the likeliest diagnosis is a slowly growing brain tumour. A stroke will have similar signs, but will start SUDDENLY.

120
Q
  1. A 50 year old man complains of drooping of his left eyelid, so that his left eye is almost closed. When you hold his left eye open, he complains of double vision, and the left pupil is larger than the right one - whats dx?
A

Ptosis + mydriasis
CNIII supplies eye-lid muscles so some ptosis possible.

PNS fibres hitchhike along outside CN111 affected = mydriasis

CN 111 palsy split into 2 catgories:
1. Surgical - compression from outside = PNS dysfunction.

  1. Medical - ischaemia/diabetes = motor component of CN111 being affected = pupil down + out
121
Q

A 12 year old boy presents with a flu-like illness, fever, headache, vomiting, tremor of the left side of the body and weakness of his left leg.

Examination of the lower limbs reveals wasting of the muscles of the left calf with absent reflexes on that side - whats DX

A

LMN lesion.

Polio - Poliovirus infection is usually asymptomatic and when symptomatic the most common presentation is with a minor GI illness. This patient has acute flaccid paralysis (AFP), or paralytic poliomyelitis, which is the hallmark of major illness. This can rarely progress to bulbar paralysis and respiratory compromise.
Paralytic poliomyelitis presents with decreased tone and motor function, as well as reduced tendon reflexes and muscle atrophy of the affected limb.

122
Q

A 82 year old lady visits her GP complaining she has
noticed a change in her voice, explaining it sounds
particularly croaky. On further questioning, she also reports
some problems with eating food, and finds soft foods like
yoghurt easier to manage. The patient has been taking
omeprazole for the last 20 years. Which of the following
steps is most important for the GP to take?
A. Discontinue omeprazole
B. Organise a Chest X-Ray
C. Organise an Upper GI Endoscopy
D. FBC
E. Organise a Barium swallow

A

The age, progressive dysphagia, soft foods are easier to eat + change in her voice suggest tumour is more likely.

Dx - Is OGD + biopsy is the most applicable investigation.

RFs - ETOH - sq 
Smoking - sq 
GoRD - adeno 
Barets - adeno
Achalsia - SQ + adeno
123
Q

A 53-year-old man presents with weight gain and low
mood.
On examination, you notice purple stretch marks on his lower
abdomen and proximal muscle wasting. Further, visual field
testing reveals a bitemporal hemianopia.
Which of the following investigations is unique to Cushing’s
disease?
A. Failure to supress the HPA axis on low-dose
dexamethasone suppression test
B. Elevated serum glucose
C. CT abdomen reveals bilaterally enlarged adrenal
glands
D. Elevated 24-hour urinary cortisol
E. Elevated petrosal sinus central to peripheral
venous ACTH ratio

A

Elevated petrosal sinus central to peripheral
venous ACTH ratio - inferior
petrosal sinus sampling can be used to calculate the central to
peripheral ACTH ratio. A ration greater than 2:1 is suggestive
of Cushing’s disease.

Cushings disease is ACTH producing pituitary adenoma.

Cushing’s syndrome includes cusings disease and other causes.

The test specific for a pituitary adenoma = excess ACTH is Elevated petrosal sinus central to peripheral
venous ACTH ratio.

Low dose dexamethasone - failure of ACTH supression in all causes of cushings syndrome - so not unique test.

124
Q
Which of the following is associated with an increased
serum transferrin level?
A. Haemochromatosis
B. Myelofibrosis
C. Iron-deficiency anaemia
D. Anaemia of chronic disease
E. beta-thalassaemia major
A

Iron-deficiency anaemia

Reduced serrum Iron
Increased TIBC
Increased transferrin
Reduced ferritin

Transferrin binds Fe+ in the blood. There is an increase to try and promote Fe2+ in the blood so compensatory increase.

125
Q

An 18 year old girl with a past medical history of anorexia
nervosa is brought to A&E by her mother after she told her
she had taken a box of 16 paracetamol tablets 2 hours ago,
intending to end her life. She has no symptoms other than
mild nausea and lethargy. What is the next most appropriate
step in management?
A. IV Naloxone
B. Gastric lavage
C. Wait until 4 hours post ingestion and measure
paracetamol levels
D. N-acetylcysteine
E. Reassure and discharge

A

Answer C – Although the treatment for a paracetamol
overdose is N-acetylcysteine, the question has asked for the
next most appropriate step, making option C the answer. The
patient has taken the paracetamol only 2 hours ago and so will
likely be asymptomatic. It is therefore important to first check
the patient’s paracetamol levels 4 hours post ingestion to
determine firstly if that patient has truly overdosed (often this
may be a cry for help) and secondly how much Nacetylcysteine
is needed.

126
Q

A 33-year-old obese woman complains of tiredness. She
has recently given birth to a healthy baby boy and is enjoying
being a mother. However, she is becoming more reliant on her
partner for support as she always feels exhausted and often
becomes depressed. The patient has a poor appetite and often
does not finish her meals, despite this she has gained 5 kg in
the last 2 weeks. The most likely diagnosis is:
A. Post partum depression
B. Eating disorder
C. Hyperthyroidism
D. Hypothyroidism
E. Occult malignancy

A

Fatigue, poor
appetite and weight gain = hypothyroidism.

Postpartum depression, occult malignancy [malignancy of uncknown origin] and
an eating disorder. However, none of these would result in
weight gain at the same time, more likely weight loss. This
makes hypothyroidism the best answer.

127
Q

A 55 year old man is brought to see his GP by his wife, as
she has noticed him slurring his words in the last few months,
even drooling from time to time. On questioning, the man
adds he also have some problem swallowing foods, both
solids and liquids. On examination, the GP notices wasting of
the tongue and fasciculations. The patient’s gag reflex is
normal. Which of these is the most likely diagnosis?
A. Myasthaenia Gravis
B. Motor Neuron Disease
C. Stroke
D. Multiple Sclerosis
E. Parkinson’s

A

Motor Neuron Disease

LMN lesion w/bulbar sx

  • bulbar sx myasthenia gravis or MND
  • LMN Sx suggests MND
128
Q

A 59 year-old man suffering from autoimmune hepatitis
attends the GP for a check up. Which of the following is the best marker of liver function?
a) ALT (Alanine Transaminase)
b) AST (Aspartate Transaminase)
c) GGT (Gamma glutamyl transferase
d) INR (International normalised ratio)
e) Serum vitamin K level

A

INR (International normalised ratio)

Due to reduced synthesis of co-agulation factors

129
Q

A 51-year-old man is brought to A&E having collapsed at
home. An ECG is performed, which reveals a regular narrow
complex tachycardia with absent P waves. He is unconscious
on admission and peripheral cyanosis is observed. BP: 84/56
mm Hg; HR = 136 bpm.
What is the most appropriate management option for this
patient?
A Vagal manoeuvres
B IV adenosine
C Verapamil
D DC Cardioversion
E IV beta-blocker

A

The first step in
the management of SVT involves determining whether the
patient is haemodynamically compromised. As this patient is
unconscious with peripheral cyanosis, this is suggestive of
haemodynamic compromise. Therefore, this patient should
undergo DC cardioversion.

130
Q

A 65-year-old lady attends her GP complaining of hearing
loss. She described her hearing as getting worse over the last
few months, but does not report any pain in her ears. Weber’s
test does not lateralise to a particular ear, whilst Rinne’s test is
negative in both ears. Which of these is the most likely cause
of the lady’s hearing loss?
A. Neurofibromatosis type 2
B. Foreign body
C. Otosclerosis
D. Otitis Media
E. Neurofibromatosis type 1

A

Rinne’s negative - means bone conduction is louder than air conduction i.e conductive hearing loss.

Weber’s dosen’t lateralise meaning hearing loss in both ears.

Conductive hearing loss:
B. Foreign body - [unilateral]
C. Otosclerosis
D. Otitis Media [unilateral]

∴ answer = C.

131
Q

A 60 year old male presents to A&E with a sudden onset
painful, cold leg. You suspect acute limb ischaemia. Which of
the following would suggest that the limb is not viable?
A Absent peripheral pulses
B Pain
C Loss of sensation
D Very cold leg
E Pallor

A

The loss of sensation is a key feature that the limb tissue is no longer viable.

It presents with a
sudden onset painful leg and may feature the 6 Ps of acute
limb ischaemia: painful, pale, pulseless, perishingly cold,
paraesthesia and paralysis.

132
Q
A nine year old boy has three days of right eye redness following the onset of cold symptoms five days ago. There is a white discharge from the eye that causes it to be difficult to open in the morning. He has not had any fevers and has no eye pain or visual changes. His younger brother had similar symptoms one week ago.
Which is the most likely diagnosis?
A. Allergic reaction
B. Blepharitis
C. Conjunctivitis
D. Trauma
E. Uveitis
A

Conjunctivitis [VIRAL]

Conjunctivitis - bacterial or viral or allergic

  • RED EYE + SWELLING OF EYELID [CHEMOSIS],
    FOREIGN BODY SENSATION
    PHOTOPHOBIA
    DISCHARGE

Bacterial:

  • Unilateral
  • Yellow discharge
  • Blurry Vision
  • URETHRITIS/VAGINAL DISCHARGE [STIS = COMMON CAUSE BACTERIAL CONJUNCTIVITIS]

VIRAL

  • BILATERAL
  • CLEAR DISCHARGE
  • NORMAL VISION
  • FLU LIKE SYMPTOMS

ALLERGIC - ITCHING/SNEAZING/SEASONAL ELEMENT

Uveitis - anterior or posterior 
Anterior AI 
- PAINFUL, OCCULAR HYPERAEMIA
- Blurry Vision 
- INCREASED LACRIMATION/PHOTOPHOBIA 

Posterior - Infective

  • Painless
  • Blurry Vision
  • FLOATERS +Scotomas
133
Q

Treatment of post-MI bradycardia?

A

Transient vagal activation resulting in sinus bradycardia is common after
myocardial infarction and may be symptomatic, as in this case, with
hypotension and probably reduced cardiac output. It usually responds to
one or more doses of atropine - muscarinic antagonist. (C).

Stimulant drugs like adrenaline are not only
ineffective in this situation but also dangerous, as they increase the risk of
ventricular arrhythmias.

134
Q

A 69-year-old man presents with confusion. His wife reports he has become
increasingly depressed and confused in the last month. Prior to this, he had been
complaining of pain in his right arm and abdomen but he refused to visit his GP.
The patient has brisk reflexes, reduced skin turgor, sunken eyes and an x-ray of his
right arm shows lytic lesions. The most appropriate management is:
A. Calcitonin
B. Non-steroidal anti-inflammatory drugs (NSAIDs)
C. Intravenous bisphosphonates
D. Stem cell transplant
E. Intravenous saline

A

[36] This patient is most likely suffering from hypercalcaemia due to an
underlying condition of untreated multiple myeloma. The patient must first be stabilized against the
effects of hypercalcaemia. Intravenous saline

135
Q

A 23-year-old woman is brought into accident and emergency after collapsing at
her office. She admits having been stressed and had stayed up all night preparing
for a presentation she gave this morning. She describes sitting at her desk and
seeing multicoloured circles of light in her right visual field then waking in the
ambulance with an oxygen mask on. She feels tired, achy and confused. Her
colleague who witnessed the event tells of his fright as he saw her collapse and start
jerking both arms and legs. What best describes her seizure?
A. Tonic–clonic seizure
B. Generalized seizure
C. Grand mal seizure
D. Simple partial seizure with secondary generalization
E. Pseudoseizure

A

This woman and her witness give a good description of a simple partial
seizure (focal visual symptoms) which generalized into a tonic–clonic
seizure (D).

Partial seizures -
Simple concious
Complex unconcious

  • Strange sights/smells
  • Jerking movements - can progress to other muscle groups Jacksonian March
  • Todd’s paralysis
  • Memory disturbance [deja vu]

Tonic-clonic: LOC - muscle stiffening - jerking /twitching- deep sleep + wake up

Tx - partial seizures - carbamazepine

Tonic clonic - Carbamazopine/lamotrigine

136
Q

A 59-year-old man, who has completed five cycles of chemotherapy for metastatic
colorectal carcinoma, presents to accident and emergency complaining of feeling
generally unwell. On examination, he is pyrexial at 38.9°C and there are crepitations
in the right lung base. What is the most urgent investigation?
A. Full blood count
B. Chest x-ray
C. Urine microscopy, culture and sensitivity
D. Blood cultures
E. CT abdomen

A

Patients who become pyrexial while receiving chemotherapy must be
managed with the diagnosis of neutropenic sepsis in mind. This is an
oncological emergency. An urgent full blood count (A) is thus required
to see if the patient is neutropenic (<1.0 × 109 mmol/L).

137
Q

A 45-year-old man presents to accident and emergency with back pain. He works
as a builder and the pain started after he had moved a cement mixer. On presentation,
he is in considerable distress and unable to walk. He has not passed urine or opened
his bowels since the incident. On peripheral neurological examination of the lower
limbs, power is reduced throughout due to the pain. Sensation is preserved except
for around the perineum. On digital rectal exam, there is poor anal tone. What is
the most likely diagnosis?
A. Spinal cord compression
B. Cauda equina syndrome
C. Nerve root compression
D. Bony injury
E. Muscular strain

A

The clinical features described in the case in this question of
inability to open bowels, inability to urinate, reduced tone of digital rectal
exam and a saddle anaesthesia, indicate that the patient has prolapsed a
disc into the cauda equina

Cauda equina - saddle anasthesia/perineal ansthesia
+ LMN signs

Spinal chord compression + cauda equina present with :

Back pain + bowel/bladder + sensory/motor dysfunction

Bowel - incontinence [reduced tone of anal sphincter]
Bladder - incontinence/retention
ED

Sensory - numbness
[perineal numbness - cauda equina]

Motor: weakness to paralysis

138
Q

A seven-year-old girl is brought to accident and emergency by her mother because
of a nose bleed that keeps on bleeding despite pressure and ice-packs. Petechiae
and ecchymoses can be seen on examination and the mother reports the child has
recently recovered from a throat infection. You suspect the patient is suffering from
immune thrombocytopenic purpura and organize tests to measure platelets (Plt),
bleeding time (BT), prothrombin time (PT) and partial thromboplastin time (PTT).
Which of the following is the most appropriate result?
A. PT: prolonged; PTT: prolonged; BT: prolonged; Plt: decreased
B. PT: normal; PTT: normal; BT: prolonged; Plt: decreased
C. PT: normal; PTT: normal; BT: prolonged; Plt: normal
D. PT: normal; PTT: prolonged; BT: prolonged; Plt: normal
E. PT: normal; PTT: prolonged; BT: normal; Plt: normal

A

B In immune thrombocytopenia a reduced number of platelets causes an
increased bleeding, as is evident in this patient (B).

Result (A) reflects
disseminated intravascular coagulation
PT: prolonged; PTT: prolonged; BT: prolonged; Plt: decreased

139
Q

next most appropriate investigation of someone presenting with heamaturia in urine dip ?

A

Rule out transient causes 1st like UTI - so do MSU.

Then investigate other causes if negative like hypertension + bladder cancer.

140
Q

What are features of Good pastures?

A

anti-GM antibodies
renal failure
Haemoptysis

141
Q

What is indicative of Von willebrand

C. PT: normal; PTT: normal; BT: prolonged; Plt: normal
D. PT: normal; PTT: prolonged; BT: prolonged; Plt: normal
E. PT: normal; PTT: prolonged; BT: normal; Plt: normal

A

(D) reflects Von Willebrand’s disease - VWF stabilises factor 8 - so prologued APTT defect in intrinsic pathway

PT: normal; PTT: prolonged; BT: normal; Plt: normal

Note difference between haemophilia and VWF is increased bleeding time is seen in WF as defect is primary haemostasis.

In haemophilia defect is in secondary haemostasis.

142
Q

A 75-year-old man with known heart failure is likely to have had a
myocardial infarction. He is now haemodynamically stable on the ward. A
12-hour troponin level is raised.
Which investigation will confirm your diagnosis?
A 24-hour ECG
B Exercise ECG
C Troponin level at 48 hours
D Nuclear studies of the myocardium
E None of the above

A

Radionuclide myocardial perfusion imaging will show reduced uptake @ infracted areas.

143
Q

Which of the following symptoms would NOT be caused by complex
partial seizures?
A Déjà-vu
B Fumbling or rubbing, slightly odd limb movements
C Shooting/electric shock-like pains in the limbs
D Smell and/or taste hallucinations
E Subtly impaired conscious level

A

E Subtly impaired conscious level

Partial seizures

  • Memory impairement [deja-vu]
  • Strange sights smells before
  • Todds paralysis
  • Jerking movements that can progress to other muscle groups [Jacksonian march]

Shooting pains into the limbs, depending on the distribution,
sounds typical of a radiculopathy, and “electric-shock” sensations sounds
like “Lhermitte’s sign”,

144
Q

A 22-year-old man with a family history of psoriasis attends clinic
complaining of lower back and buttock pains on and off for the past 6
months. This is worse in the mornings and associated with stiffness.
Which of the following findings on magnetic resonance imaging (MRI)
would be first to appear and thus diagnostic of early ankylosing spondylitis?
A Bamboo spine
B Fusion of the sacroiliac joints
C Osteophyte formation
D Sacroiliitis
E Syndesmophyte formation

A
  • Sacroiliitis

Hx - lower back pain + sacroiliitis

  • progressive loss of movement.
  • Asyymetric arthritis
  1. Enthesitis - leads to reactive new bone formation + calcified vertebral fusion + reactive new bone formation [Syndesmophyte formation]
On X-ray:
- Blurring of the VERTEBRAL RIMS. 
- THEN SYNDESMOPHYTE 
- THEN eventually bamboo spine. 
-
145
Q

A 35-year-old homeless man diagnosed with new tuberculosis 1 month
previously and started on treatment presents with a loss of colour vision.
Which of the following medications is responsible?
A Ethambutol
B Isoniazid
C Pyrazinamide
D Rifampicin
E Streptomycin

A

Ethambutol

Side effects of TB medication:

  • Ethambutamol: E = eyes
  • Pyrazinamide - pyramids - god’s - gout
  • Isoniazid - [feeling isolated, on the periphery - peripheral neuropathy]
  • RifamPICIN - [rifampissing oRANGE PISSIN - piss people off and INDUCE rage - induce CYP450]

Standard quadruple therapy, which will be given for the first 2
months in new tuberculosis, before continuing the isoniazid and rifampicin
for a further 4 months.

146
Q

Which of the following patients is most likely to have a cerebellopontine
angle tumour?
A A 30-year-old woman with unilateral lower motor neuron facial weakness
progressing over days and covered in café au lait spots
B A 40-year-old woman with insidious onset unilateral deafness, on Weber’s
testing sound is louder ipsilaterally
C A 50-year-old man with bilateral facial weakness, nasal voice and
swallowing difficulty progressing over 3 months
D A 50-year-old woman with insidious-onset unilateral deafness, on Weber’s
testing sound is louder contralaterally, and with some ipsilateral reduced
facial sensation
E A 65-year-old man with insidious-onset mild bilateral sensorineural
deafness and no other neurological abnormality

A

Cerebellopontine tumour = deafness + facial weakness [CN VII + 8]

Other focal localising synbdromes

  • cavernous sinus
  • SPHENOID WING MENINGIOMA -
    PINEAL REGION
    PITUITARY FOSSA
147
Q
A 65 year old man presents with a 7 month history of vague epigastric pain. The pain is made worse immediately after eating and is relieved by vomiting. The patient avoids spicy foods and has lost 9kg in weight. On examination there is midline epigastric tenderness - Dx? 
A. Pancreatitis
B. Duodenal ulcer
C. Gastric carcinoma
D. GORD
E. Gastritis
F. Pancreatic carcinoma
G. Gastric ulcer
H. Acute cholecystitis
I. Oesophagitis
A

Weight loss is one of the most common presenting symptoms in patients with gastric cancer. Epigastric pain is present in about 80% and may resemble that of a gastric ulcer.
On examination there may be the presence of a left supraclavicular node (Virchow’s node), a periumbilical nodule (Sister Mary Joseph’s nodule) or a left axillary nodule (Irish node). A mass may be palpable in advanced disease. More proximal tumours can present with dysphagia. Acanthosis nigricans, a black velvety rash, may also be present.

Suspected gastric malignancy is an urgent upper GI endoscopy with biopsy of the lesion.

148
Q
55 year old woman who complains of dysphagia is found to have a 10cms in width &amp; 5cms in height bilateral symmetrical non-tender nodular mass in the front of her neck. Investigations show that she is euthyroid.
All Answer Choices
A. Reassure &amp; explain why no active management necessary
B. Thyroxine
C. Sialogram
D. Technetium thyroid scan
E. Carbimazole
F. Full blood count &amp; Paul Bunnell
G. Excise for biopsy
H. 
Amoxicillin
A

The bilateral mass in the last part of this question is a multinodular goitre which does not need a biopsy. The patient is euthyroid. If the nodule was unilateral, then a biopsy is essential to establish or exclude malignancy. However, her dysphagia needs to be investigated with an upper GI endoscopy.

149
Q

A 63 year old man who has recently had chemotherapy for bowel cancer is suffering from diarrhoea.

A

Codeine phosphate is an opiate and will kill two birds with one stone by offering effective pain relief and treating the diarrhoea. Codeine is used for mild to moderate pain, diarrhoea and as an antitussive.

150
Q
62 year old man with enlarging pulsatile mass in left groin. 3 days previously he had had a coronary angiogram.
A. Lipoma
B.  Strangulated femoral hernia
C. Abscess
D. Aneurysm
E. False aneurysm
F. Reactive lymph node
G. Inguinal hernia
H. Folliculitis
I. Lymphoma
J. Sebaceous cyst
A

This is a false or pseudoaneurysm as it does not involve all layers of the arterial wall. This has resulted from arterial trauma during the angiogram. Most cases of false aneuryms are as a result of iatrogenic trauma. A haematoma has formed between the breached layers and the remaining intact artery, causing the lump, which is pulsatile as blood rushes through the artery.

151
Q
66 year old woman with right leg ulcer dressed by district nurse for 2 months. On examination there is a tender warm lump in the right groinA. Lipoma
B.  Strangulated femoral hernia
C. Abscess
D. Aneurysm
E. False aneurysm
F. Reactive lymph node
G. Inguinal hernia
H. Folliculitis
I. Lymphoma
J. Sebaceous cyst
A

Reactive lymph node

The leg ulcer indicates that there is local inflammation. Reactive lymphadenopathy is common when there is infection or inflammation.

152
Q
A 50 year old man with a 1 month history of progressive abdominal distension preceded by increased tiredness, shortness of breath on exertion &amp; weight loss of 10kg. There is a non-tender irregular mass in the right iliac fossa.
A. Heart failure
B. Nephrotic syndrome
C. Budd Chiari syndrome
D. Primary liver tumour
E. Liver cirrhosis
F. Tuberculous peritonitis
G. Primary biliary cirrhosis
H. Carcinoma of caecum with peritoneal secondaries
I. Carcinoma of the ovary
J. Secondary liver tumours
K. Bacterial peritonitis
A

The weight loss of 10kg, fatigue and non-tender irregular RIF mass point to caecal carcinoma. Right sided colorectal cancer tends to present with anaemic symptoms. Almost 90% are anaemic at diagnosis. The progressive abdominal distension indicates the presence of peritoneal secondaries, which causes vague symptoms. Treatment in this case for a cancer that has become widely metastatic will be palliative.

153
Q

A 60 year old man has suprapubic pain on standing and difficulty emptying his bladder with blood at the end of the stream.
A.
Localised prostate cancer

B. Acute prostatitis
C.
Bacterial cystitis

D. Advanced prostate cancer
E.
Gram negative septicaemia

F. Bladder calculus
G. Hydronephrosis
H. Benign prostatic enlargement

A

F. Bladder calculus
There is pain on standing and difficulty urinating because the calculus falls and blocks the bladder outflow. Blood comes at the end of the stream from urine that has passed the damaged area of urothelium. Patients tend to present with suprapubic pain, haematuria and obstructive symptoms. E
Examination may reveal suprapubic tenderness. The first test to order is urinalysis. A non-contrast CT abdomen is also indicated to look for the stone.

154
Q

Drug side effects?
Beta-blockers e.g atenolol
Tricyclic antidepressants e.g amitriptyline, imipramine

A

Beta-blockers - Cold toes and fingers. [beta receptor mediated vasodilation of cutaneous vessels]

TCAs have antimuscarinic properties and thus have the classic atropine-like side effects including dry mouth, constipation, mydriasis and blurred vision.

155
Q

A 65 year old man undergoes elective excision of a rectal carcinoma and primary anastomosis. He is left with a stoma.

What type of stoma?

A

Loop Illeostomy to divert faecal flow allowing primary anastamosis to heal.

[loop = temporray]

156
Q

A 25 year old man with a history of ulcerative colitis, deteriorates and requires and emergency operation to treat the disease.

What type of stoma?

A

Sub-total colectomy + end illeostomy

[most of colon removed + end illeostomy] in severe UC.

157
Q

A 72 year old woman has a disseminated malignancy of the colon. She is suffering from abdominal distention and increasing constipation. She has a procedure before going on to palliative care.

What type of stoma?

A

Loop colostomy is used to divert faecal flow to bypass an obstruction usually.

158
Q

Low calcium, Low phosphate, High PTH, High ALP

High calcium, Normal phosphate, Slightly low PTH , High ALP

Normal calcium, Normal phosphate, Normal PTH, Normal ALP

A. Hypercalcaemia of malignancy
B. Osteomalacia
C. Primary hyperparathyroidism
D. Paget's disease
E. 
Myeloma

F.
Sarcoidosis

G. Osteoporosis
H.
Hypoparathyroidism

I. Immobility

A

Osteomalacia = vit D def = low cacium absoprtion so [low serum calcium]
Secondary parathyroidism
High ALP to increase blood calcium

High calcium, Normal phosphate, Slightly low PTH , High ALP - hypercalcaemia of malignancy

Normal calcium, Normal phosphate, Normal PTH, Normal ALP - Osteopaenia = normal ranges.